NR602 Final Exam LATEST 2022-2024 (4 DIFFERENT VERSIONS)/NR 602 FINAL EXAM CHAMBERLAIN COLLEGE.

Nr602 final exam questions and answers
Nr602 final exam questions
nr 602 final exam pediatrics
nr 602 final exam 2021
nr 602 week 4 midterm exam
nr602 week 7 quiz
nr 602 syllabus chamberlain

Step 1 Asthma approach-Intermittent

symptoms 2x or less per week
asymptomatic and normal PED
requires SABA 2 days/week
no interference with normal activities
brief exacerbations

nighttime symptoms 2x or less a month
lung fx- FEV>80% predicted

Step 2 Asthma Approach-Mild persistent

Symptoms >2 x a week, less than once per day
requires SABA more than 2days/week, no more than once a day
exacerbations may affect activity
nighttime symptoms 3-4x a month
FEV> 80% predicted

Step 3 Asthma Approach-Moderate Persistant

daily symptoms
daily use of SABA
some limitations
2x or more per week exacerbations
nighttime symptoms more than 1x per week, not nightly
FEV >60% but <80%

Step 4 Asthma Approach-Severe Persistent

continual symptoms
requires SABA multiple x a day
extremely limited activity
nighttime symptoms 7x a week
FEV <60%

Tx of asthma

Stepwise approach
step 1: SABA PRN
step 2: low dose ICS
Step 3: low dose ICS+ LABA or medium dose ICS
step 4: Medium dose ICS+LABA
Step 5: high dose ICS+ LABA
Step 6: High dose ICS+LABA + corticosteroid

Step 6 Asthma Approach

Bulbar/palpebral conjunctival infection

May be unilateral or bilateral

Leukocoria

abnormal appearance of a white film in the pupil; immediate referral to pediatric ophthalmologist warranted
Causes: retinal detachment, cataract, retinal dysplasia, newborn retinoblastoma

Visual screening in children

At least once between ages 3-5 y/o according to USPSTF

AOM

RF: genetics, males, Native American, siblings, low economic status, ages 6mo-3y, winter, supine bottle feeding, daycare, tobacco smoke

S/S of AOM

erythema, otalgia, bulging TM, absent cone of light

Dx of AOM

Audiometry, tympanometry, possible lateral neck xray to r/o mass

TX of AOM

uncomplicated: supportive with tylenol/ibuprofen; watchful waiting 48-72 in 6m-2y/o; <5 benzocaine otic drops
1st line antx: amoxicillin 80-90mg/kg/day Q12 x 10days
if allergy to PCN- augmentin, cefuroxime

Bacterial rhinosinusitis

Preceded by URI-typically worsens after 5-7 days- not resolved in 2 weeks

Sx of bacterial rhinosinusitis

Purulant nasal congestion, drainage, facial pain, headache, fever
No imaging required- if no improvement refer to ENT

Bronchiolitis

Usually caused by RSV
wheezing present
<2 y/o
other causes; influenza, adenovirus, rhinovirus

S/s of bronchiolitis

Increased work of breathing, prolonged expiration, grunting, retractions, nasal flaring

Croup sx

Low grade fever, URI symptoms, barking cough, inspiratory stridor can occur

Croup dx

Made from symptoms

Croup tx

Glucocorticoids possibly
0.6mg/kg-1mg/kg
humidified air
bronchodilators

Lead poisoning

Inactivated heme synthesis by inhibiting insertion of iron-leads to microcytic hypochromic anemia

Source of lead poisoning

Lead based pain

Those at risk for lead poisoning

Children 2-3 y/o
summer months

Lead poisoning testing

Children with Medicaid need lead level @ 12 months and 24 months-capillary finger stick with venous sample as f/u
AAP recommends 6-9-12-18-24 mo as well as 3-4-5-6 y/o

Lead levels

<5 is normal
>69 requires chelation

Genu varum

Bow legged as a result of uterine position- normal finding up to 3y/o

Legg-Calve-Perthes Disease

Avascular necrosis of femoral head- epiphyses associated with trauma, synovitis

Legg-Calve-Perthes Disease RF

Associated with low birth weight, socioeconomic status, or white race

Legg-Calve-Perthes Disease s/s

Onset of painful limp of thigh, knee, or hip worse with activity, not relieved by rest
restricted by abduction and rotation of affected hip

Legg-Calve-Perthes Disease Tx

Abduction brace or long leg cast

Congenital Hip Dysplasia s/s

Thick fold asymmetry, leg length inequality, walking children- painless limp

Congenital Hip Dysplasia Dx & Tx

Positive Barlow maneuver, ortolani or Allis sign
US for <4 months, X-ray AP of pelvis >4 months
Tx: refer to orthopedist, pavlik harness, child should be seen weekly to prevent skin breakdown, necrosis

Toxic trait synovitis

Unilateral inflammation arthritis; acute onset; decreased ROM extension and internal rotation; painful hip, crying at night; common in boy 3-6 y/o

Toxic trait synovitis Dx and Tx

Dx: WBC with leukocytosis, increased ESR, hip xray normal
To: BR, NSAIDs, non WB

Impetigo

Superficial layers of the skin
Nonbullous or bullous
nonbullous= honey-colored crusts on lesions
caused by group A streptococcus, S.aerous or MRSA
occurs more in summer months, low socioeconomic class

Impetigo exam

Lesions on hand, face, neck, extremities or perineium; regional lymphadenopathy

Impetigo treatment

Topical antx if superficial, nonbullous or localized to one area
bacitracin
neomycin
polymyxin B
Widespread infection again S. Aerous= Augmentin, cephelexin, dicloxacillin, cloxacillin for 7-10 days

Cellulitis

In children often periorbital, perivaginal, perinatal, or buccal

Cellulitis hx

Recent URI, fever, pain, malaise

Cellulitis exam

Buccal cellulitis-blue/purple tinged= h.influenzae

Cellulitis Dx

CBC, BCx if child appears toxic, has fever, or is <1

Cellulitis tx

IV antx
if streptococcal= PCN, if allergy 3rd generation cephalosporin
if staphylococcus=bactrim if child >2 months; doxy if child >8 and < 45 kg
MRSA suspected= clindamycin
H.influenzae=augmentin x 10 days

Folliculitis

Infection of hair follicle; s. Aerous most common

furnucle

Deeper infection of hair follicle involving the deep dermis

Hot tub folliculitis

Pseudomonas aeruginosa

Dx of folliculitis & furnucle

Grain stain and cx

Management of folliculitis and furnucle

Warm compresses, benzoyl peroxide
if superficial- topical antx-erythromycin or clindamycin
if severe or widespread- cephalexin or dicloxacillin

Paronychia

Infection around fingernail/toenail
S.aerous #1 then streptococcus or pseudomonas

Paronychia tx

Systemic oral antx if acute infection present
candida-nystatin
if purulant-loosen cuticle with blade

Candiasis

Tx: thrush-oral nystatin QID
if resistant to tx: oral fluconazole
skin diaper rash-nystatin, ketonazole

Tinea captitis

ringworm of the scalp
diffuse fine scale without obvious hair loss
discrete area of hair loss with broken hairs (black dot ringworm)
trichophyton tonsurans and microsporum canis-most common organisms
African American boys most common

Image: Tinea captitis

tx of tinea capitis

griseofulvin ultramicrosize once or twice daily for 6-8 weeks, take with fatty food to increase absorption
shampoo with econazole or ketonazole in addition

tinea corporis

ringworm
found on non hairy part of body

Image: tinea corporis

tx for tinea corporis

topical antifungals miconazole or clotrimazole 1-4 weeks BID

Tinea cruris

jock itch
4-6week of antifungals

tinea pedis

3-6 weeks of antifungals

onychomycosis

fungal infection of the nail
typically with T.rubrum or candida
exam= opaque, white, silvery nail that becomes thick/yellow
seldom symmetrical
tx: oral terbinafine, fluconazole, itraconazole

Image: onychomycosis

tinea versicolor

a fungal infection that causes painless, discolored areas on the skin
occurs on the trunk
more in adolescents, warm weather, immunocompromised
exam= hypopigmented or hyper (salmon colored to brown) with raindrop or guttate appearance
Tx: selenium sulfide lotion or shampoo for 2-4 weeks
older adolescents can use ketonazole

Image: tinea versicolor

herpex simplex

HSV type 1-oral mucosa, pharynx, lips
caused by herpes labialis-cold sore/fever blister
HSV-2 neonatal

Image: herpex simplex

Herpes simplex exam

HSV-1= gingivostomatitis- grouped vesicle that ulcerate and form white plaques on mucosa, gingiva, tongue, chin, labial folds
halitosis present
herpes labialis-cluster of small clear vesicle with erythematous base usually only on one side of the mouth
HSV-2= grouped vesicopustules and ulceration with edema, primary lesions on vaginal mucosa, labia, perineum, penile shaft

Herpes simplex dx & tx

Dx: viral cx= gold standard for diagnosis
Tx: acyclovir 20-40mg/kg/dose 5x daily for 5 days

Herpes zoster

shingles
more common in adolescents
exam- 2-3 clustered groups of macules and papules-progress to vesicle on the a red based, vesicles become pustular, rupture, and crust
Tx: antivirals not usually recommended unless immunocompromised.

Image: Herpes zoster

Molluscum contagiosum

small firm, pink, papules 1-6mm, itchy
occurs on face, axillae, antecubital area, trunk
tx: usually disappear on own
if pain occurring curettage, salicylic acid

Image: Molluscum contagiosum

Warts

caused by HPV virus
most common on extremities but can occur on face, scalp, and genitalia
verruca vulgans-common wart
verucca plantaris-plantar warm
condylomata acuminata- on genital
mucosa-cauliflower appearance
TX: watchful waiting, no treatment if asymptomatic

Image: Warts

pediculosis

head lice
Tx: OTC permethrin 1%
1st step- apply permethrin or pyrethrin
2nd step- remove nits, comb hair
3rd step-cleanse environment

Image: pediculosis

Scabies

itching that is worse at night-S shaped burrows
webs of fingers, fold of wrist, arm pits, forearm
TX: permethrin 5%, repeat in 1 week

Image: Scabies

Drug eruption

exanthematous reaction-morbilliform
measles like rash
Most common drugs: PCN, cephalosporin, sulfonamide antx, NSAIDS, antifungals,
typical onset 1-2 week of starting new med
Tx: D/c drug, antihistamines

Image: Drug eruption

Erythema multiforme

skin disorder resulting from a generalized allergic reaction to an illness, infection, or medication

Image: Erythema multiforme

Pityriasis Rosea

Presents with a herald patch, Christmas-tree pattern.

Image: Pityriasis Rosea

Psoriasis

chronic skin condition producing red lesions covered with silvery scales

Image: Psoriasis

Psoriasis treatment

■ Topical STEROIDS

■ Topical RETINOIDS (TAZOROTENE)

■ TAR preparations (PSORALEN drug class).

keratosis pilaris treatment

Removal of built-up keratin; lotions, creams, or ointments; topical steroids

Image: keratosis pilaris treatment

lichen striatus

1. An uncommon, unilateral and linear eruption of erythematous, flat topped and scaly papules that may extend the length of a limb

2. Age of onset 3-14 years, Females > males

Etiology: unknown

Tx:
1. Spontaneous involution occurs by 9 months duration

2. Topical and intralesional steroids

Image: lichen striatus

Mongolian spots

areas of deep bluish-gray pigmentation most commonly on the sacral aspect of a newborn

Image: Mongolian spots

amblyopia

-a dimness of vision or the partial loss of sight, especially in one eye, without detectable disease of the eye
-results in reduced or permanent loss of vision
-Findings: squinting, abnormal vision, pain and/or headaches, fatigue, dizziness, developmental delay
-usually responds to 2 hours of daily patching

strabismus

“lazy eye”
referral after 4 months of age
“good eye” is occluded

blapharoptosis

drooping of the upper eyelid
if vision is compromised surgery is performed

Retinoblastoma

tumor arising from a developing retinal cell (a congenital, malignant tumor)
all infants should have red reflex exam before d/c from hospital
findings: strabismus, decreased visual acuity, unilateral or bilateral leukoria

Image: Retinoblastoma

Conjunctivitis

H. influenzae, strepococcus, pneumonae, or moraxalla
typically occurs December-April
bacterial-unilateral
viral-bilateral

conjunctivitis in newborn

usually caused by
chlamydia-clear mucoid exudate
gonorrhea-purulant exudate

Conjunctivitis in newborn Dx and Tx:

cx, gram stain
Tx: Saline then erythromycin ointment

Bacterial conjunctivits

erythema, itching, burning, mucopurulant d/c
Dx: R/O URI, pharyngitis, AOM
Tx: neonates-erythromycin
> 1 year fourth generation fluoroquinolone (moxifloxacin)

Viral conjunctivits

caused by adenovirus, HSV, varicella, herpes zoster
Sx: fever, unilateral photophobia, bilateral tearing, erythema
Tx: refer to optho if HSV or photophobia present
cool compresses 3-4 x daily

Image: Viral conjunctivits

Blepharitis

inflammation of the eyelash follicles
bilateral
Findings: swelling, erythema of eyelids, flaky debris upon wakening, gritty/burning in eyes
Tx: Bacitracin or erythromycin ointment
purchase new makeup, cool compresses

Image: Blepharitis

Uveitis

acute onset of pain, red eye, photophobia, blurred/decreased vision, excessive tearing
Dx: slit lamp exam
Tx. refer to optho, corticosteroids typically used

Image: Uveitis

Otitis media

Findings: bulging TM, decrease translucency otorrhea, white or yellow TM
Tx: 1st line amoxicillin
if allergy to PCN-azithromycin

Image: Otitis media

Otitis media with effusion

children often asymptomatic
dull, bulging TM
amber colored TM
watchful waiting for 3 months.

Image: Otitis media with effusion

cystic fibrosis

genetic disorder maifested by COPD, GI disturbances, and exocrine dysfunction

CF manifestations

Neonate-meconium ileus, prolonged jaundice, hemorrhagic disease of newborn (vit K deficiency)
infancy- cough, pneumonia, FTT, steatorrhea, abdominal distention
childhood- poor weight gain, constipation, sinusitis, poorly controlled asthma, digital clubbing
adolescents- diabetes, osteoporosis, chronic pansinusitis

CF Dx

newborn screening
sweat testing- sweat chloride >60 is indicative of CF
LFTs are abnormal

CF tx

-inhaled dornase- decreases mucosal viscosity
-CFTR modulators
-high dose ibuprofen and azithromycin 3x a week
-pancreatic enzyme replacement therapy (PERT) with every meal and snack
-at age 10 yearly glucose tolerance test

Microcytic anemia (iron deficiency anemia)

dietary iron primarily absorbed in the duodenum
HGB screening at 12 mo for anemia
lead poisoning often comorbid diagnosis
RF: premature, >6 mo BF without iron supplementation
alternative diets, low socioeconomic status

characterized by: microcytic hypochromic RBC, low or normal MCV, increased RDW, low ferritin, high TIBC

Findings of IDA

infants/toddlers-restless irritable, PICA, pagophagia (desire to ingest ice), anorexia, developmental delays

Tx of IDA

iron supplementation 3-6mg/kg/day in 2-3 divided doses

SCA

Findings: fatigue, anemia, pain crises, bacterial infections, dactylitis (swelling of hands and/or feet), priapism, splenic sequestration

SCA Dx

HGB 6-10
HCT 20-29%

SCA tx

pain, hydration, oxygen utilization, other care
refer to pediatric hematologist
should receive PCN prophylaxes until 5th birthday or received two doses of PPSV23

Anemia of prematurity

-MC anemia in premature, low birth weight infant
-low RBC production and short RBC lifespan
-normocytic normochromic anemia
retic low (v hemolytic, sickle cell – retic high)
-normal WBC, plt count
-normal total bili (v hemolytic)
-tx: iron supplemnts, blood transfusion

Galeazzi sign

when inspecting the thighs and gluteal fold of the hips w/ knees flexed, a shortening of the femur w/asymmetrical skin folds is positive for DDH.

Image: Galeazzi sign

Barlow maneuver

place your index and middle finger over the greater tronchanter. Gently push both knees together at midline downwards.
Positive:”Clunk” sounds or palpating trochangter being displaced by the index/middle finger

Image: Barlow maneuver

Ortolani maneuver

check hips for congenital dislocation, done until 1 yr old, should be smooth with no sounds
, abnormal= feels like a clunk as head of femur pops back into place- positive ortolani sign

Image: Ortolani maneuver

annular ligament displacement

Nurse maids elbow
Tx: supination and flexion or pronation
do not perform if epitrochanter tenderness is present
palpable click is successful reduction

Scoliosis

abnormal lateral curvature of the spine
idiopathic- unknown etiology
infantile- 0-3 years
juvenile- 3-10 years
adolescent- 11 years and older
Hx: family hx, painless curvature
Dx: AP and lateral views standing of spine
Management: refer to orthopedist, brace or surgery, observation for <20 degrees

Osgood Schlatter Disease

microtrauma in deep fibers of patellar tendon at insertion to the tibial tuberosity
seen in chidlren who wetn through grown spurt in last year
boys>girls

Osgood Schlatter Sx and tx

increase pain with and after activity
bilateral
recent physical activity
pain reproduced when extending knee against resistance
focal swelling
full ROM
TX: avoid activity, ice/heat, stretching, NSAIDs, neoprene sleeve over knee

leukemia

bone marrow is replaced with blast cells
most common form of childhood cancer
Findings: anemia, pale, repeated infections, fever, wt loss, bleeding episodes
Dx: CBC-thrombocytopenia, anemia presents
peripheral smear-malignant cells
bone marrow exam

Pediatric migraine

more than 5
duration 2-72 hours
pulsating quality
unilateral in adolescents
bilateral in young children
moderate intensity with activity
nausea/vomiting
photophobia

tension type headache

10 episodes occurring <1 day a month
last from 30min-7days
bilateral
pressing quality; mild moderate
no nausea/vomiting
no photophobia

Chronic tension headache

– >15 per month for >3 mo
-Mild to mod intensity
-Non-pulsating pain
-Not aggravated by ADL
-No vomiting
-Can have light, sound sensitvity or nausea

Red flags with headaches

vomiting that relieves headache
pain that awakens child
first/worst headache
occipital region with neck pain
Tx: MRI if ICP symptoms present, neuro exam abnormal, dizziness/double vision present

tx for pediatric headaches

NSAIDs first line

Epilepsy

chronic brain disorder characterized by recurrent seizure activity
Dx: CBC, LFTs, blood glucose
LP if child >6 months
EEG standard dx
MRI if EEG reveals abnormality

Epilepsy tx

refer to pediatric neurology
common medications: levetiracetam, lamotrigine, topiramate, valproic acid
Defer DTap immunization unless consultation with pediatric neurologist

febrile seizures

6-60 months typically
38 degrees Celsius
Dx: EEG if complex febrile seizures

cerebral palsy

paralysis caused by damage to the area of the brain responsible for movement
symptoms appear within first few years of life
Exam= orthopedic-scoliosis, contractures, dislocated hips
neuro- asymmetrical or abnormal DTR, persistent tonic neck or Moro reflexes
macrocephaly or microcephaly

CP Dx

MRI

Problems with CP

feeding/eating problems
seizures
vision problems: amblyopia, strabismus,
constipation
drooling
inadequate nutrition

CP medications

baclofen, tizanidine, diazepam
minimize contractures and spasticity
botulinum toxin A

Tourette syndrome

motor and verbal tics that persist > 1 year
simple tic-blinking, twitching hands, shoulder shrugging, tongue thrusting
Tx: psychotherapy

Depression tx

CBT and SSRI first line

Bipolar disorder

1-2% of children <18 meet diagnostic criteria
early onset irritability; continuous rapid cycling
Sx: severe mood changes, inflated self esteem, decrease need for sleep, racing thoughts
depression typically first symptoms
Tx: refer to behavior health
medications-mood stabilizers, lithium, valproate, risperidone

ODD

occurs in early childhood 3-7 years old, most likely 8
behavior that is negative, defiant, disobedient, and hostile

Rheumatic fever

A bacterial infection that can be carried in the blood to the joints
autoimmune response to an anaerobic gram positive coccus, group A streptococcus (GAS)
involves joints, heart, CNS (Sydenham chorea) and subcutaneous tissue
usually occurs 2 weeks after infection

RF Dx

echocardiogram to confirm
positive throat cx
rising antistrepolysin O titer
elevated ESR, CRP, leukocytosis, prolong PR on EKG

RF findings

revised Jones criteria
abdominal pain, malaise, pericardial pain, rapid sleeping pulse rate, tachycardia
polyarthritis- knees elbows, wrist-most common
carditis
erythema marginatum-bright pink blanching papules or macules that spread on trunk/extremities, nonpruritic
subcutaneous nodules over bony prominences

RF tx

benzathine PCN- drug of choice
if allergy to PCN-erythromycin
anti-inflammatory therapy
NSAIDs

primary prevention of RF

tx GAS within 9 days of onset (sore throat)
oral PCN or amoxicillin

Kawasaki disease

acute febrile disease-leading cause of acquired heart disease in children
asian/pacific islander most at risk

Kawasaki disease findings

conjunctival hyperemia, erythematous rash, edema hands/feet, unilateral lymph nodes

kawasaki dx

diagnosis of occlusion
CBC, cmp, ESR, cRP

kawasaki disease management

evoke anti-inflammatory response
prevent coronary thrombosis by inhibiting platelet aggregation
minimize long term heart risk factors by exercise, healthy diet, smoking prevention
IVIG therapy
high dose of aspirin for anti-inflammatory
ECHO initial then repeat at 2 weeks then 6-8 weeks
live viruses should be pushed until 11 months after IVIG

GERD

Sx: ‘heartburn’, chest pain, pain in sternum, regurgitation with or without vomiting, weight loss, poor weight gain, dysphagie, respiratory disorders
Dx: nonradiological tests; CBC, UA, stool for occult blood, h.pylori testing,
esophageal pH monitoring-gold standard to diagnose reflux
multichannel intraluminal impedance-measures reflex useful in making diagnosis in children with respiratory disorders and reflux

GERD managment

histamine 2 receptor agonists
PPIs
buffering agents-antacids

UTI

dx: if ill child-catheterization should be used not bagged specimens
causes cx with >100,000 in clean catch, >50,000 in catheterization
Tx: infant 2-24 months should have antx 7-14 days
bactrim 8-12mg/kg
amoxicillin if <3 months 20-30mg/kg/day in 2 doses
augmentin
cephelexin
pyridium for >6 years old
children <2 year should have renal/bladder US when antx have been completed.

Wilms tumor

a rare type of malignant tumor of the kidney that occurs in young children
firm, smooth mass in abdomen or flank
most common between 2-5 years old
increase frequency in African American

Wilms tumor findings

increased abdominal size, palpable mass, pain, fever, dyspnea, vomiting, weight loss

Wilms tumor dx and tx

dx: chest/abdominal xray, abdominal US, UA with hematuria
CBC, reticulocyte, liver/renal studies
tx: refer to pediatric cancer center

Testicular Torsion

twisting of the spermatic cord causing decreased blood flow to the testis
can occur after physical exertion, trauma
uncommon before 10y/o

testicular torsion findings

unilateral scrotal pain with nausea/vomiting
fever is minimal or absent
intermittent testicular pain
minor trauma

testicular torsion exam

ill appearing male, resisting movement
swellingof scrotum with redness, warmth, tenderness
“blue dot” sign-subtle blue mass visible
elevation of tests increases pain
cremasteric reflex absent on side of torsion

testicular torsion dx and tx

UA normal, doppler US
surgical intervention

gastroenteritis

children >2 four or more x in 24 hours
can last up to 14 days
Dx: ordered if more serious infection present
stool exam, cx, pH, electrolytes, CBC
Tx: antidiarrheal not unless child is > 3 years old; loperamide
probiotics

Cholesteatoma

cystlike mass composed of epithelial cells and cholesterol occurring in the middle ear; may be associated with chronic otitis media
Exam: chronic OM, purulant otorrhea
vertigo/hearing loss, pearly white lesion present on or behind TM
TX: surgical-refer to otolaryngologist

Image: Cholesteatoma

head injury/concussion/TBI

most common cause of trauma in pediatrics
Dx: use CDC acute concussion evaluation (ACE)
Exam- vital signs, careful oral exam, GCS, CDC Heads up for concussion symptoms
CT if GCS 3-12
CT if LOC >1 min, AMS, neuro deficits
Brain trauma indicator-blood test to evaluate TBI

TBI management

closed injury with no LOC- monitor
closed injury with minor/brief LOC- monitor closely, if unable to at home, may need hospitalization
moderate injury with worrisome symptoms-hospitalization with neurology consult and critical care

pinworms

Enterobius vermicularis
Tiny nematode
Lays eggs around the anus, causing local itching especially at night
Treatment with albendazole and mebendazole-repeat in 2 weeks
positive tape test reveals eggs

turner syndrome

A chromosomal disorder in females in which either an X chromosome is missing, making the person XO instead of XX, or part of one X chromosome is deleted.

short stature expected
typically nonverbal
strabismus
hearing issues
monitor BP (HTN)
annual thyroid screen
increased risk for celiac
increased risk for hyperlipidemia, cardiac defects
supplemental estrogen therapy

Down syndrome

careful review of newborn screen for hypothyroidism
ongoing cardiac evaluations
monitor for growth using syndrome specific chart
eye exam for cataracts
ongoing hearing exams
monitor for OSA
increase risk for leukemia
increased risk for celiac disease

Acyanotic cardiac disease

left to right shunting-increased pulmonary blood flow
atrial septal defect
ventricular sepal defect
patent ductus arteriosus

obstruction to ventricular outflow
coarctation of the aorta
pulmonary stenosis
aortic stenosis

Cyanotic Cardiac Anomalies

right to left shunting
decrease pulmonary blood flow
tetralogy of Fallot
tricuspid atresia
hypoplastic left heart syndrome

mixed blood flow
transposition of the great vessels
truncus arteriosus
critical pulmonic stenosis

diabetes insipidus

inability to synthesize and release vasopressin
kidneys cannot concentrate urine-excessive loss
s/s: abrupt onset, polyuria, nocturia, enuresis, intense thirst
tx: DDAVP oral or intranasal BID

Diabetes type 1

most common in patients <20 y/o
autoimmune damage to beta cells of pancreas
s/s hyperglycemia, polyuria, polydipsia, polyphagia, weight loss, urine ketones
labs: Hgb A1C, CMP, CBC< lipids, fasting, and 2 hours post prandial, TSH
pediatric endocrinologist

ASCCP recommended management for an ASCUS pap result with high risk hpv 16 cotest in 26 year old no abnormal paps
colposcopy

What is included in the well women of an adolescent?
health history
menses
gynecologic and pregnancy issues
psychosocial
abuse drugs alcohol use
physical exam
screening tests
immunizations

When does ACOG pap smear start
21 years old done every three years

ACOG pap for over 30 yrs old
every five years with HPV test completed

ACOG when can you stop pap smear
can stop at 65 if negative history for 10 years or if they had a hysterectomy with no history of cervical cancer

Bethesda pap smear report reads LSIL is what classification?
CIN 1

A single Pap smear reading of ASCUS in a patient negative for HPV infection should have what as follow-up?
routine screening

a female patient is 35 years old. she has never had an abnormal PAP smear has regular screening since age 18. If she has a normal PAP smear with HPV testing today, when should she have the next cervical cancer screening?
5 years

A young sexually active is having her first pap smear what would you explain?
pap smear detects cervical cancer

26 year old pap smear shows negative and positive HPV what do you do?
repeat HPV and pap

what is not part of criteria for older women to cease pap smears?
over 55 year old

no menarche in a 15 year old with no medical problems and Tanner stage 1 with a uterus
primary amenorrhea

No menses for at least 6 months
secondary amenorrhea

female athlete triad
disordered eating, amenorrhea, osteoporosis

labs for female athlete triad
HCG
serum prolactin
serum TSH,FSH, LH

if amenorhea lasts longer than 6 months do what?
bone density test

treatment plan for female athlete triad
increase caloric intake, decrease exercise
prescribe ca and vit d 1200/1500 daily and vitamin e 400 iu daily

complications of female athlete triad
osteopenia/osteoporosis (stress fracture)
myocardial atrophy, arrhythmia, hypotension, brady
hypoglycemia, dehydration, electrolytes
lanugo, telogen effluvium hair loss zerosis dry skin, infertility, low body mass, cachexia, respiratory failure

purpose of progesterone challenge
endogenous estrogen

primary and secondary amenorrhea originates in hypothalamus
sheehans syndrome

symptoms of breast cancer ( early)
None
Non-painful mass (occasionally pain)
Eczematous nipple (Paget’s)
Serosanguinous (bloody) nipple discharge

late symptoms of breast cancer
skin or nipple retraction
axilary lymhadenpathy
breast enlargement
redness, edema, brawny induration
peau d’ orange
pain
fixation of the mass to the skin or chest wall

very late findings of breast cancer
ulceration
supraclavicular lymphadenopathy
edema of the arm, bone, lung, liver, brain or other distant metstases

Follow up for breast cancer patients

  1. Bilateral mammogram 6 months after complete radiation post-lumpectomy; Mammogram yearly after
  2. Contralateral mammogram annually post-mastectomy (entire breast + pectoralis major fascia)

PE every 3-6 months for 3 years then annually

fibrocystic breast disease
the presence of single or multiple benign cysts in the breasts

signs and symptoms of fibrocystic breast disease
occurs 2 weeks before the onset of mensus and worst right before the menstrual cycle. resolves after menses starts often in women in their 30’s. breast lumps are rubbery and mobile to touch

Tx for fibrocystic breast disease
Once a benign diagnosis or normal findings have been established by biopsy or on clinical or imaging findings, simple reassurance will provide many patients with adequate relief.

For those patients who still seek treatment, symptomatic relief by avoiding trauma and by wearing a bra with adequate support can be very helpful. The role of caffeine consumption in the development and treatment of fibrocystic change has never been proven; however, many patients report relief of symptoms after abstinence from coffee, tea, and chocolate. Similarly, observational studies have suggested that low-fat diets can provide some relief. The data regarding the utility of vitamin E supplementation and evening primrose oil are controversial. Mild analgesics such as acetaminophen and nonsteroidal anti-inflammatory drugs (NSAIDs) can be used to relieve breast pain. For more symptomatic women, danazol and tamoxifen have been found to be effective, although their significant side effects have limited their acceptability and utility.

Fibroadenoma
benign breast lesion 15-40 yrs old

risk factors for fibroadenoma
multiple fibroadenomas associated with rate cancer syndromes like mafficci syndrome, Cowden syndrome, carney complex early ETOH consumption

findings with fibroadenoma
painless, firm or rubbery mass with well defined borders, freely mobile, changes with size may occur with pregnany or menstrual cycle, lesions under 5 cm, considered to be giant fibrodenomas no nipple discharge

Dx of fibroadenoma
mammo, us to differentiate fluid filled to solid mass, fine needle aspiration biopsy, open biopsy

Intraductal papilloma
benign tumor within the ductal system of the breast, one of the heterogeneous group of lesions encompassed by the term benign breast disease seen in females 30-50

risk factors for intraductal papilloma
multiple papilloma’s which increases the risk for breast cancer

dx studies for intraductal papilloma
mammo, us mri, galactography, fine needle or core biopsy

a pap smear result of atypical squamous cells of undetermined significance rule out high grade squamous intraepithelial lesions will require which procedure next?
colposcopy

pap smear is ASCUS what is next?
follow up pap smear

Bartholin’s glands obstruction symptoms
pain, tenderness, dyspareunia, walking with adducted thighs

assessment findings of Bartholin’s glands obstruction
firm labia mass or cyst, erythema, induration, edema of

treatment for bartholins gland obstruction
warm baths
Augmentin for e coli and strep
staph or MRSA clindamycin
re assess in 7-10 days
drainage of the infected cyst through marsupialization or WORD catheter

BMI below 18.5
underweight

BMI 18.5-24.9
normal

BMI 25-29.9
overweight

BMI >30
obese

annual screening for chlamydia and gonorrhea at increased risk
every 3-6 months

annual screening for pharyngeal gonorrhea throat at increased risk
every 3-6 months

annual testing for HIV, syphilis and HBsAG
test frequently

screen pregnant woman at first prenatal visit for
HIV, chlamydia, syphilis, hepatitis B

testing for pregnant women after chlamydia and gonorrhea treatment
within 3-4 weeks after tx

retest for chlamydia and gonorrhea (not for cure)
3 months

PID tx
Rocephin 250 IM plus
Doxy bid x 14 days
with or without flagyl x 14 days

PID symptoms
pelvic lower ab pain and one of the following cervical motion tenderness, uterine or adnexa tenderness

testing for PID
endometrial bx, transvag US, laprascopy

cervix disorders
fusion of mullerian ducts to form cervix and corpus uteri

Cervical agenesis
an absent uterine cervix with a normal uterine corpus and normal vagina

rx of cervical agenesis
suppression of menstruation with continuous combined estrogen progesterone pills may improve the pain related complaints.

dx of cervical agenesis
ultrasonography, MRI, laparoscopy

Incomplete Mullerian Fusion
complete failure of fusion of the Mullerian ducts results in duplication of preproduction structures.

Failure of Resorption
complete spectate uterus

Diethylstilbestrol (DES)
cervical abnormalities and are at increased risk for infertility

laceration of the cervix
cervical cerclage, precipitous labor, vacuum extraction, nulliparity and episiotomy are associated with increased risk for clinically significant lacerations

perforation of the cervix
may occur during sounding of the cervix, cervical dilation, dilation of cervix, insertions of radioactive sources or conization of the cervix or during self inducted abortion with sharp objects

ulceration of the cervix
from pressure necrosis from vaginal pessary. or from uterine prolapse when the cervix protrudes through the vaginal introitus

cervical stenosis
narrowing or obstruction of the cervical canal caused by an acquired condition

Pyometra
infection of the uterus

Dx of pyometra
biopsy to rule out endometrial carcinoma

tx of pyometra
1) OHE w/ pre-surgical stabilization is paramount

2) anti-biotic therapy for 2 weeks post-op

annular detachment
extremely rare
devitalized and torn uterus
and expelled during labor

complications of cervical injuries
hemorrhage
hypovolemic shock

Mongolian spots
areas of deep bluish-gray pigmentation most commonly on the sacral aspect of a newborn

lead poisoning
test at 12 months of age

cystic fibrosis
A genetic disorder that is present at birth and affects both the respiratory and digestive systems.

Traumatic Brain Injury (TBI)
A traumatic insult to the brain capable of producing physical, intellectual, emotional, social, and vocational changes.

sickle cell anemia
a genetic disorder that causes abnormal hemoglobin, resulting in some red blood cells assuming an abnormal sickle shape

iron deficiency anemia
anemia caused by inadequate iron intake

Anemia of prematurity
-MC anemia in premature, low birth weight infant
-low RBC production and short RBC lifespan
-normocytic normochromic anemia
-retic low (v hemolytic, sickle cell – retic high)
-normal WBC, plt count
-normal total bili (v hemolytic)
-tx: iron supplemnts, blood transfusion

Thalassemia
inherited defect in ability to produce hemoglobin, leading to hypochromia

otitis media
inflammation of the middle ear

Tourrette Syndrome
involuntary, spasmodic, twitching movements; uncontrollable vocal sounds; and inappropriate words

cerebral palsy
paralysis caused by damage to the area of the brain responsible for movement

Pediatric UTI
-bowel or bladder dysfunction: withholding maneuvers, incontinence, constipation
-vesicoureteral reflux
-tx with 3rd generation cephalosporin: cefixime, cefdinir, ceftibuten for 3-5 days if afebrile and 10 days if febrile
-if febrile refer to urology
renal bladder ultrasound (RBUS) for all infants 2-24 months with first febrile UTI

ADHD
a psychological disorder marked by the appearance by age 7 of one or more of three key symptoms: extreme inattention, hyperactivity, and impulsivity

dyslexia
impairment of the ability to read

odd
oppositional defiant disorder

bipolar disorder
A mood disorder in which the person alternates between the hopelessness and lethargy of depression and the overexcited state of mania.

Depression
A prolonged feeling of helplessness, hopelessness, and sadness

Pinworms

  • Come out of the anus at night to lay their eggs, Shower in the AM and underwear should be changed at this time with the child standing.
  • The child doesn’t need to stay out of school, good hand hygiene contains the spread of pinworms.
  • Pinworm paddle lab test
  • Wash bed linens in hot soapy water.

congenital heart defects
structural abnormalities caused by the failure of the heart to develop normally before birth

Kawasaki disease
(inflammation of blood vessles, hence the strawberry tongue) causes coronary artery aneurysms.

RSV
respiratory syncytial virus

croup
an acute respiratory syndrome in children and infants characterized by obstruction of the larynx, hoarseness, and a barking cough

genu varum
bow legged

Legg-Calve-Perthes Disease
degeneration of femoral head due to avascular necrosis. disease is self limiting and has 4 phases: condensation, fragmentation, re-ossification and remodeling. presents with pain, decreased ROM, antalgic gait, positive Trendelenburg sign. primary treatment focus is to relieve pain and maintain femoral head in proper position.

Scoliosis
abnormal lateral curvature of the spine

Osgood-Schlatter disease
inflammation or irritation of the tibia at its point of attachment with the patellar tendon

febrile seizures
Seizures that result from sudden high fevers, particularly in children.

testicular torsion
twisting of the spermatic cord causing decreased blood flow to the testis

Willms tumor
a rare type of malignant tumor of the kidney that occurs in young children

Turner Syndrome
A chromosomal disorder in females in which either an X chromosome is missing, making the person XO instead of XX, or part of one X chromosome is deleted.

Down Syndrome
a condition of intellectual disability and associated physical disorders caused by an extra copy of chromosome 21.

hip dysplasia
abnormal development of the pelvic joint causing the head of the femur and the acetabulum not to be aligned properly

gastroenteritis
inflammation of the stomach and intestines

GERD
gastroesophageal reflux disease

cystocele
protrusion of the bladder

Causes of cystocele
childbirth
chronic constipation
violent coughing
heavy lifting
overweight
age
hysterectomy

sx of cystocele
feeling of fullness
pressure in the vagina
increased discomfort when straining, coughing, bear down
feeling of incomplete empty
repeated bladder infection
pain or urinary leak during sex
bulge of tissue into vaginal opening

prevention of cystocele
kegels
prevent constipation
avoid heavy lifting
avoid wt gain

descent or prolapse of the bladder
when speculum is retracting the posterior vaginal wall ask to strain

dysmenorrhea
painful menstruation

tx of dysmenorrhea
NSAIDs, OCPs are TOC;

primary dysmenorrhea
Results from increased prostaglandin production during the luteal phase of the menstrual cycle, when estrogen and progesterone levels decline

secondary dysmenorrhea
painful menstruation that begins during adult life, usually as a consequence of a pelvic disorder

Primary amenorrhea
the failure to begin menstruating at puberty

secondary amenorrhea
the absence of menstruation after a period of normal menses for 6 months

interstitial cystitis
painful bladder syndrome
not STD
chronic condition causing bladder pressure
sometimes pelvic pain

sx of interstitial cystitis
nocturia, dysuria, urgency, suprapubic pressure

risk factors of interstitial cystitis
over 30

Tx of interstitial cystitis
pelvic PT
NSAIDS
TCA
antihistamine
pentosane polysulfate sodium

linchen sclerosis
severe itching
dryness/irritation
labia adhered
benign chronic inflammatory process

causes of lichen sclerosis
vitamin A deficiency
autoimmune
excess enzyme elastase
decreased activity of 5 alpha reductase

Sx of lichen sclerosis?
itching
vulvar pain
pain with sex
white lesions
starts with erythema and edema over vulvar skin then white plaques and hyperkeratosis then uniting of plaques and itching

Dx of lichen sclerosis
vulvar biopsy
fixed labia
adhesions

Tx lichen sclerosis
high potency topical steroids
avoid tight undies
cleanse daily with soap and dry
oral antihistamine at night

risk factor of lichen sclerosis
vulvar carcinoma

Lichen Simplex Chronicus
lichenification –long-term atopic dermatitis — repetitive scratching/rubbing

well-circulated plaque

tx: stop the itch-scratch cycle; dressing/topical steroids, tar, anti-histamine
should resolve in 4-6 weeks

Appearance of lichen simplex chronicus
diffusely red to red brown

Metronidazole
inhibits DNA synthesis and rapidly bactericidal
drug of choice for gardenerella vaginalis, trich and BV

adverse effects of metronidazole
do not take with ETOH (antabuse effects) HA, N/V
metallic taste and prolonged use can cause peripheral neuropathy

Molluscum contagiosum
What virus causes small pink benign wartlike tumors and is associated with HIV-positive patients?

Sx of molluscum contagiosum
round firm
painless bumps
size pinhead to eraser
small induration
may be seen on genitals
lower ab and inner thighs and will spread sexaully

Nabothian glands
mucus filled cyst on surface of cervix
small white raised bump
no tx

Small, pale yellow, raised, and rounded areas are visualized on the surface of the cervix. You should?
chart as nabothian cyst

PCOS
polycystic ovarian syndrome
oyster ovaries
sclerocystic
smooth pearl white surface without indentations
persistent anovulation enlarged polycystic ovaries
secondary amenorrhea
oligomenorrhea
obesity
hirsutism
infertility
r/t hypothalamic pituitary dysfunction and insulin resistance

Labs for PCOS
elevated serum androgen
increased LH to FSH ratio
lipid abnormalities
insulin resistance

Tx of PCOS

  • weight loss

-OCP
comiphene citrate for infertility
need progesterone

PID
pelvic inflammatory disease
endometriosis
salpingitis
tube ovarian abscess
pelvic peritonitis
organism N gonorrhea and C trachomatis

Dx of PID
clinical dx confirmed by CMT, uterine tenderness, or adnexal tenderness

NAAT from cervical swab determines pathogen

definitive dx laparoscopy.

Prevention of PID
treat for chlamydia and gonorrhea
abstain from intercourse during treatment

S/S of PID
abdominal pain and pelvic pain
purulent vaginal discharge
nausea
HA
hypoactive bowel sounds
extreme tenderness with bimanual
cervical motion tenderness
uterine tenderness
adnexal tenderness

Labs for PID
leuks, ESR/CRP, trasvaginal sonography/ MRI
tubal hyperemia

complications of PID
peritonitis, adhesions, bacteremia, infertility

tx for PID
broad spectrum abx
commonly rocephin 250mg x1
doxy 100 bid x14 days with or without metronidazole 500 mg BID x14 days.
if pregnant must be IP with parenteral tx
close follow up

pylenophritis
bacteria in urine culture/ bacterial infection of kidney

Sx of Pyelonephritis
Fever Chills Flank pain CVA tenderness (costovertebral angle — tender above kidneys on back)

Dx of pyelonephritis
ua with culture

tx of pylenophritis
bactrim 14-21 days and antipyretics for fever

complications of gonococcal and chlamydial genitourinary infection in women include all of the following except
acute pylenophritis

rectocele
hernia of the rectum into the vagina

Sx of rectocele
soft bulge of tissue in the vagina
difficult BM
sensation of rectal pressure
incomplete emptying after BM
sexual concerns

Causes of rectocele
constipation/ strain
chronic cough
heavy lifting
overweight
childbirth
age

prevention of rectocele
kegels, prevent constipation, avoid heavy lifting, cough avoid wt gain

Skene’s glands
glands located on the anterior wall of the vagina, around the lower end of the urethra

STD risk factors
Multiple partners, unprotected sexual intercourse, high frequency of sexual intercourse, high risk behavior, young age at first intercourse, and unusual sexual practices. All sexually active patients should be screened for STDs.

herpes simplex
cold sores

prevention of herpes simplex
consistant condom use because viral shedding can occur in asymptomatic periods and can lead to transmissions

Tx for Herpes Simplex?
acyclovir, valacyclovir, famciclovir
500 mg daily

Sx of herpes simplex
tingling, itching, burning, sores, flu-like symptoms, urinary problems, eye infections

Dx of Herpes Simplex
cell culture and PCR

Tx of Herpes simplex
systemic antiviral acyclovir 7-10 days for first episode
suppressive 200 mg daily BID

Chancroid
caused byHaemophilus ducreyi
transmitted via sexual contact or on hands that have touched the lesion
reportable disease

Sx of chancroid
-soft, painful genital ulcer
-painful inguinal LAD

Dx of chancroid
culture for H. ducrei

Tx of chancroid
Azithromycin 1g po x1
ceftriaxone 250 mg IM x1
cipro 500 mg PO BID x3 days
erythromycin 500 mg PO TID x7 days
personal hygiene, clean with soap and water
sitz bath

Syphilis
an STD that attacks many parts of the body and is caused by a small bacterium called a spirochete
reportable disease

Syphilis stages
primary, secondary, latent, tertiary

Primary syphilis
mucus mem 10-90 days later

secondary syphlis
2 wks -6 mo after primary lesion then generalized cutaneous eruption of secondary may appear

Latent syphilis
the third stage of syphilis, which may last for years, during which symptoms disappear although the person is still infected

prevention of syphilis
condom
wash with soap and water after sex
screen people at high risk

Sx of primary syphilis

  • painless chancre
  • large LNs

Sx of secondary syphilis
flu like prodrome w. variable skin rash on palmar/plantar surfaces; mucous patches, lymphadenopathy, malaise, anorexia, alopecia, arthralgias

sx of latent syphilis
resolution

Dx of syphilis
Dark field micorscopy (motile spirochetes),
VDRL/RPR (rapid, cheap, sens only 60-75% in 1°),
FTA-ABS (sens & specific, used as 2° dx test),
T. pallidum particle agglutination test (TPPA, sens & spec similar to FTA-ABS but easier to use)

tx of syphilis
Penicillin G

Chlamydia trachomatis
negative whiff with mucopurulent and positive clue cells. gold standard test is NAT, not culture
reportable

Prevention of Chlamydia
-abstinence
-condoms
-monogamy
-ID and treat sexual contacts
-testing

Sx of chlamydia
same as gonorrhea except
-mucopurulent cervicitis
-abd pain
-post coital bleeding

Dx of Chlamydia
Usu clinical;
Cx = gold standard;
Urine tests (PCR or ligase chain reaction) = rapid means of detection, DNA probes & immunofluorescence take 48-72 hrs;
Gram stain = PMNs but no bacteria (intracellular)

Tx of chlamydia
Doxycyclin 100 mg PO BID x 7 days,
Or Azithromycin 1 g PO x 1 day;
Use Erythromycin in pregnant pts;
Tx sexual partners & maintain low threshold to tx for N. gonorrhoeae;
LGV serovars require prolonged tx for 21 days

complications of chlamydia
salpingitis, PID, perihepatitis

Gonorrhea
A sexually transmitted bacterial disease caused by a gonococcus bacterium that causes inflammation of the genital mucous membrane, burning pain when urinating, and a discharge
reportable

prevention of gonorrhea
-abstinence
-condoms
-monogamy
-ID and treat sexual contacts

Sx of gonorrhea
-asymptomatic
-vaginal discharge
-inc frequency
-dysuria

  • NOT ‘MUCOPURLENT’

Dx of gonorrhea
Gram stain & cx = gold standard;
Nucleic acid amplification tests can be sent from penile/vaginal tissue or from urine

complications of gonorrhea
PID, infertility, ectopic pregnancy, reactive arthritis

Tx of gonorrhea
Ceftriaxone IM x 1 OR cipro or cefixime
Concurrent tx for chlamydia as well since chlamydia much more common

HSV-1
herpes simplex virus type 1

HSV-2
genital herpes
cervical lesion
first vesicular then ulcers
after intial infection virus continues to reside in nerve cells for life

HPV
genital warts
skin to skin contact transmission

Prevention of HPV infection
abstinence
condom use
treating affected partner@ same time

trich
caused by flagellated protozoan
mobile

Prevention of Trichomoniasis
Usual STI preventions, including condoms
decrease # of sex partners
valvular hygiene

sx of trichomoniasis

  • FEMALES:
  1. Vaginitis: w/ frothy green secretions, foul odor and intense irritation strawberry appearing cervix
  • MALES:
  1. Urethritis
  2. Prostatis
  3. Sterility
  4. OFTEN ASYMPTOMATIC

dx of trichomoniasis

  • Vaginal/Prostatic Wet Mounts: Pyriform, FLAGELLATED TROPHOZOITES

tx of trichomoniasis
metronidazole or tinidazole
2g po single dose PO

Candida albicans
yeast infection

Dx of Candidiasis
KOH

tx of candidiasis
Fluconazole

BV
bacterial vaginosis

risk factors for BV
-tight fitting clothes
-mult. sexual partners
-recent abx use
-HSV-2 infection
-douching

prevention of BV
condom use
no douching

sx of BV
fishy smelling, grey, watery discharge w/ vaginal spotting
3 of 4 amsel criteria

Dx of BV
vaginal wet mount, clue cells typical

Tx of BV
Metronidazole 500 mg PO x 7 days
metronidazole gel 0.75% x5 days
clinda cream 2% x3 nights or tinidazole po x2 days

Hep B
caused by hep b virus
transmitted via blood with other concentration in would exudate
semen vaginal secretions
saliva
reportable disease
can cause liver failure and death

risk factors for hep b
IV drug use, homo/bisexual lifestyle, unprotected sexual contact, blood transfusion, tattoos

Prevention of Hep B
hep B immune globulin provides 3-6 mo protection and used post exposure prophylaxis in adjunct to vaccinate or in unvaccinated person from pre and post exposure requires a series
routinely given to all pregnant women

dx of hep b
blood test
igm antibody is dx

sx of hep b
asymptomatic
constitutional at first anorexia, jaundice, RUQ pain

Tx Hep B
supportive care
no antiviral drugs

Hep C
Genetic variations created during this viruses replications result in marked variety in the antigenic structure of HCV envelop proteins, thus explaining why an effective immune response is difficult and why vaccine development is not possible yet.

prevention of hep c
no vaccine available
reducing transmission and chronic liver disease
patients cannot donate blood
dont share razors or toothbrushes

sx of hep c
asymptomatic or mild illness

Dx of Hep C
nucleic acid PCR

Tx of hep C includes
Combination of pegylated interferon alpha and the antiviral drug ribavirin for a period of 24-48wks depending on genotype

HIV/AIDS
the virus that causes AIDS, spread through bodily fluids rather than casual contact or airborne

HIV transmission
blood, semen, vaginal fluid, breast milk

Prevention of HIV
abstinence, monogamy, safe sex, no needle sharing, universal precautions for medical personnel

Sx of HIV
wt loss
fever
night sweats
pharyngitis
lymphenopathy
reddened maculopapular rash
extragenital lymphadenopathy

Dx of HIV
Western Bolt (#1)
Elisa
WBC
*IF a negative result retest in 3 months.

Tx of HIV
PREVENTION
Antiretrovirals (HAART) and chemoprophylaxis
Screening for diseases/infections
Counseling
Postexposure prophylaxis (PEP) with ART

a 21 year old female presents with three 0.5 cm human papilloma virus lesions on her vulva. An appropriate treatment option for this patient would be?
podophyllin

Stein-Leventhal syndrome
aka polycystic ovary syndrome, young women, amenorrhea, infertility, obesity, hirsutism p295

dx of stein-leventhal syndrome
2 out of 3 oligomenorrhea
amenorrhea
hyperandrogenism
polycystic ovaries on US
elevated androgens
increased LH/FSH ratio
abnormal lipids
insulin resistance
trans vaginal US

Turner’s syndrome symptoms
Symptoms:

  • Can be unilateral or bilateral
  • Unilateral: can give birth to a normal infant
  • Bilateral: contains very few primordial germ cells (i.e. likely sterile)
  • migrate from yolk sac, few reach genital ridges
  • mother’s hormones stimulates the fetus’ cells
  • six months after birth, no oocytes
  • will look female, but will not menstruate at puberty

uterine prolapse
the condition in which the uterus slides from its normal position in the pelvic cavity and sags into the vagina

uterine prolapse sx
heaviness
pulling into pelvis
tissue protuding from the vagina
urinary problems
leakage and retention
trouble having a BM
feeling of sitting on small ball
sexual concerns

prevention of uterine prolapse
kegels
treat constipation
correct lifting
avoid weight gain

Tx of uterine prolapse
Proph
-Kegel exercises, weight reduction
Nonsurg
-vaginal pessary
Surg
-hysterectomy; uterosacral or sacrospinous ligament fixation

UTI
E. coli, Staphylococcus saprophyticus (young women)

Dx of UTI
urinalysis (clean catch (mid stream), catheter)
-WBC and RBC in blood
urine culture and sensitivity (dipstick)

  • (+) if 100,000 CFU, or >100 CFU from catheter or symptomatic

Tx of UTI
antibiotics
pyridum for pain
bactrim BID
macrobid x5 days
if complicated treat for 7 days or longer
cipro 500mg Bid x 7-10 days or macrobid 7-10 days if allergic to keflex

Vulvar carcinoma presentation
presents as leukoplakia; biopsy may be required to differentiate from the other causes of leukoplakia

Sx vulvar cancer
itching
mass
vulvar bleeding and pain and tumor found during pelvic exam

dx of vulvar cancer
biopsy

Tx of vulvar cancer
surgical removal of the tumor

pelvic pain
Adenosis and fibroids (from tumors in the uterine wall or submucosal or subserosal surfaces arising from the smooth muscle cells of the myometrium) are causes of what?

uterine fibroids
benign tumors composed of muscle and fibrous tissue that develop in the uterus

primary treatment of uterine fibroids
drainage of the infected cyst or abscess
marsupialization or by insertion of a word catheter
appropriate antibiotics
excision of the cyst

Leukemia
cancer of white blood cells

rheumatic fever
A bacterial infection that can be carried in the blood to the joints

illegal substance
any substance forbidden by law to be used by our society because of the dangers associated with their use or abuse

Headaches
a diffuse pain in different portions of the head and not confined to any nerve distribution area

type 1 diabetes mellitus
diabetes in which no beta-cell production of insulin occurs and the patient is dependent on insulin for survival

Epilepsy
chronic brain disorder characterized by recurrent seizure activity

Salter-Harris Classification
1- slipped
2- above
3- beLow
4- Throough
5-cRushed

Assessment of the Musculoskeletal System
-Inspect skeleton and extremities for alignment and symmetry
-Inspect muscles for size and symmetry
-Palpate bones and joints and muscles for tenderness, heat or edema
-Observe ROM
-Do passive if unable to do active
-Test muscle strength comparing sides
-Observe gait
-Smooth? Equal strides?

Galeazzi sign
when inspecting the thighs and gluteal fold of the hips w/ knees flexed, a shortening of the femur w/asymmetrical skin folds is positive for DDH.

Barlow test
Test for a hip that is dislocatable but not dislocated in infants. With infant supine and hip and knees flexed, push posteriorly in line with the shaft of femur. An unstable femoral head will dislocate posteriorly from acetabulum.

Ortonlani maneuver
reverse barlow
chunk sound is positive

Klisic Sign
finger over the greater trochanter and iliac spine line should point to umbilicus

Trendelenburg test
The patient stands and rises one foot and then the other while the doctor observes the buttocks.

Positive sign is when the buttock drops on the side that the foot is elevated indicating hip abductor weakness on the stance leg side.

IgA
immunoglobulin A
allergic anaphylaxis atopy

IgA (Berger’s)
antibody

IgA class
c
immune complex

IgA (dimer)
delayed

Adam’s Test
Forward bending test for Scoliosis

genu varum
bow legged

Genu valgum (knock knees)
This is a condition in which the knees touch but the ankles do not. Can be normal in children up to age 3.

Narakas Sensory Grading System
S0
S1
S2
S3
S0: no reaction to painfu stim
S1: reaction to painful stim
S2: reaction to touch (not light)
S3: normal

annular ligament
binds the head of the radius to the ulna

costochondritis
an inflammation of the cartilage that connects a rib to the sternum

Legg-Calve-Perthes Disease
degeneration of femoral head due to avascular necrosis. disease is self limiting and has 4 phases: condensation, fragmentation, re-ossification and remodeling. presents with pain, decreased ROM, antalgic gait, positive Trendelenburg sign. primary treatment focus is to relieve pain and maintain femoral head in proper psition.

Rheumatic disease
any disease or condition involving the musculoskeletal system

labs for children with rheumatic disease
crp
esr
serum ferrian
platelets
procalcitonin
cbc
cmp
ua

asthma levels
intermittant >80 normal between exacerbations
mild >80
mod >60 <80
severe <60

Henoch-Schonlein Purpura
IgA-anti-IgA immune complexes -> palpable purpura on buttocks/legs, IgA GN, arthritis

SCID
severe combined immunodeficiency disease

Wiskoh-Aldrich Syndrome
eczema abnormal immune system reduced clotting

Hyper IgE syndrome (Job syndrome)
Chronic skin infections

  • Persistent “cold” abscesses
  • Recurrent sinusitis
  • Eczema
  • Eosinophilia and high IgE
  • Bone defects (fractures)
  • Late/absent shedding of baby teeth
    Due to defects in Th17 function (STAT3, Tyk2, DOCK8)
    Defective signal transduction for multiple cytokines, including IL-6 and IL-23.

DeGeorge Syndrome
What is a congenital cause of hypocalcemia due to parathyroid hypoplasia, thymic hypoplasia and cardiac outflow track defect called?

Achondroplasia
A form of human dwarfism caused by a single dominant allele; the homozygous condition is lethal

chondrodystrophy
dwarfism, defective nutrition of cartilage

Prader-Willi Syndrome
Caused by a lack of genetic material in the 15 pair of chromosomes. Usually inherited from the father. The leading genetic cause of obesity. The degree of mental retardation varies, but is usually in the mild range. Students with Prader-Willi syndrome can be rigid and oppositional. They do not respond well to sudden changes in their routine. It can also result in low muscle tone, short stature, incomplete sexual development, cognitive disabilities, problem behaviors, and a chronic feeling of hunger that can lead to excessive eating and life-threatening obesity.

SHOX gene
this gene is responsible for long bone growth and is defective in pt. with Turner Syndrome

McCune-Albright syndrome
characterized by triad of fibrous dysplasia of the bone, endocrine abnormalities (such as early puberty or hyperthyroidism) and cafe-au lait spots. Condition results from an activating mutation in the G protein/CAMP/adenylate cyclase signaling pathway.

Coast of Maine border with the cafe-au-lait (irregular border)

Testotoxicosis
cause of PPP
~ testes enlarge independent of HPGA

premature thelarche
early breast development with no other hormone dependent signs (pubic hair, menses)

PKU
A human metabolic disease caused by a mutation in a gene coding for a phenylalanine processing enzyme (phenylalanine hydroxylase), which leads to accumulation of phenylalanine and mental retardation if not treated; inherited as an autosomal recessive phenotype.

MSUD (Maple Syrup Urine Disease)
Metabolic defect that affects the metabolism of branched-chain amino acids

Homocystinuria
Cystathione synthase deficiency
Lens subluxation, thrombosis, marfanoid, intellectual disabiliity
Tx: pyridoxine

Propionic acidemia
Causes buildup of valine, isoleucine, threonine, methionine, cholesterol, odd chain fatty acids (can’t be converted to methylmalonyl coA and then to succinyl CoA)

Methylmalonic acidemia
results from a defect in the isomerization of methylmalonyl coA to succinyl CoA (to enter the TCA cycle)

Ornithine transcarbamylase deficiency
Most common urea cycle disorder
X-linked recessive (other urea cycle enzyme deficiencies are autosomal recessive)
First few days of life, may present later
Excess carbamoyl phosphate converted to orotic acid

High orotic acid in blood and urine
Low BUN
Hyperammonemia symptoms
No megaloblastic anemia (vs orotic aciduria)

Citrullinemia
Argininosuccinate synthetase (ASS) deficiency. hyperammonemia, increased citrulline levels in urine/serum. can sometimes treat with arginine (stimulates NAG which may stimulate CPS 1)

Galactosemia
recessive genetic disorder; characterized by body’s inability to tolerate galactose

MCAD deficiency
medium chain acyl-CoA dehydrogenase (ACADM gene) def. → no FA β-oxidation when low blood glucose → hypoketotic hypoglycemia (can’t make ketone bodies)
AR

VLCAD Deficiency
Enzyme deficiency, thus no acyl-CoA is produced. Results in accumulation of acyl carnitine and blockage of metabolism

LCHAD deficiency
fatty acid oxidation

Leigh Syndrome
degradation of motor skills

MNGIE
progessive degerations of the muscle in the GI tract
dropping eyelids
restricted eye movements

Pearson Syndrome
Mitochondrial disease

Hereditary Orotic Aciduria
Orotic Acid is excreted in the urine because UMP synthase is defective
-pyrimidines cannot be synthesized, and therefore growth retardation occurs
-oral administration of uridine bypasses the metabolic block and provides a source of pyrimidines

Lesch-Nyhan syndrome
What X-linked recessive disorder is characterized by hyperuricemia, spastic cerebral palsy, mental retardation, and self-mutilation?

Niemann-Pick disease
sphingomyelinase deficiency results in macrophages that have soap-bubble appearance.

Zellweger syndrome
Defective peroxisome biogenesis = defective proxisomal oxidation. Inability to breakdown LC FAs due to peroxisome defect. FAs are cut short and sent to mitochondria. hepatomeg, Mental Retardation, Infantile death. Severe

Infantile Refsum disease

  • Later onset of variable initial symptoms
  • No neuronal migration defects
  • No (or less) progressive white matter disease
  • May have little or no facial dysmorphism
  • Survival variable

•Cognitive and motor delays vary from severe to moderate learning disabilities

  • Retinopathy
  • Sensorineural hearing loss
  • Many with poor growth
  • May have hyperoxluria

Adrenoleukodystrophy
impaired addition of CoA to long chain FA; FA accumulates damaging adrenal gland & white matter

Hurler syndrome
mucopolysaccharidosis, deficient a L iduronidase, accumulations of heparin sulfate and dermatan sulfate in heart, brain, liver, and other organs, progressive, hepatosplenomagaly, dwarfism, gargoyle-like facies, stubby fingers, corneal clouding, mental retardation, death by 10 years of age p57

Fabry disease
is an inherited deficiency of alpha-glactosidase A that causes accumulation of the globoside ceramide trihexoside in tissues. The earliest manifestations are angiokeratomas, hypohidrosis and acroparesthesia .Without enzyme replacement, patients typically develop progressive renal failure.

tinea pedis
athlete’s foot

tinea capitis
ringworm of the scalp

Tinea versicolor
a fungal infection that causes painless, discolored areas on the skin

pediculosis
infestation with lice

Scabies treatment
Permethrin 5% cream (Remember you have 5 fingers)
Starve mites by sealing them in a bag for about 10 days.

blepharoptosis
drooping of the upper eyelid

reinoblastoma
eye cancer

Trachoma
chronic, contagious form of conjunctivitis that typically leads to blindness

Trachoma treatment
Doxycyline or azithromycin

Hyphema
blood in the anterior chamber of the eye

Hyphema treatment
Eye protection, rest with Head of bed at 30 degree, topical beta adrenergic blockers or carbonic anhydrase inhibitors

Avoid: NSAIDS

Entropian
turning inward of the eyelid

ectropion
the eversion of the edge of an eyelid

Euryblepharon
Abnormally large eyelid opening

pterygium
thin tissue growing into the cornea from the conjunctiva, usually caused from sun exposure

epilottitis
inflammation of the epiglottis

Bronchiolitis
Inflammation of the bronchioles that usually occurs in children younger than 2 years and is often caused by the respiratory syncytial virus.

Cystic Fibrosis Treatment
PERT
High pro, high kcal, unrestricted fat, liberal in salt
Calories to cover RDA for age/sex
Supplement water sol. forms of fat sol. vitamins, especially A & E

Trisomy 21
Down syndrome

Trisomy 18 (Edwards Syndrome)
MR
Rocker bottom feet
Prominent occiput
Clenched fists w/overlapping fingers
Congenital heart disease
Horseshoe kidneys
IUGR/FTT
90% die by age 1

Trisomy 13 (Patau Syndrome)
severe cleft lip and/or palate, microphthlamia, severe mental retardation, microcephaly, holoprosencephaly, NTDs, polydactyly, rocker-bottom feet,

Monosomy X (Turner Syndrome)
produces X0 females, who are sterile; it is the only known viable monosomy in humans

Klinefelter syndrome
A chromosomal disorder in which males have an extra X chromosome, making them XXY instead of XY.

Williams Syndrome
a genetic condition characterized by mental retardation in most regards but surprisingly good use of language relative to their other abilities

Marfan Syndrome
genetic connective tissue disorder that can cause a ruptured aorta

Noonan Syndrome
PTPN11, SOS1, KRAS, RAF1 (AD)
facial features, renal malformation, bleeding disorders
Sequencing in the above order

Costello syndrome
HRAS, AD
DD, coarse, cardiomyopathy, pulmonary stenosis

Alagille syndrome
Autosomal Dominant: Bile duct paucity with cholestasis, pulmonary artery stenosis, butterfly vertebrae, triangular shaped facies

Heterotaxy
transposition, abnormal position of organs

charge
A measure of the extra positive or negative particles that an object has.

Stages of Pertussis
Stage 1: catarrhal stage, like common cold
Stage 2: paroxysmal stage—violent coughing sieges
Stage 3: convalescence stage

Jacobsen Syndrome
Deletion of tip of chromosome 11

HoltOram
syndrome
Gene/Inheritance: TBX5 ; AD 85%
de novo
Clinical Features: variable upper limb malformations
(radius, wrist, thumb); congenital
heart malformation (ASD / VSD);
cardiac conduction disease
Testing/Management: monitor heart findings with
EKG and echo
Other: exclude those who have other abnormalities (e.g. kidney, lower limbs, anus, eyes, etc.)

Neurofibromatosis
AD; pigmented neurofibromas; cafe au lait spots

LEOPARD syndrome
PTPN11, RAF1 (AD)
Lentigines, Electrocardiogram abnl, Ocular hyperteloroism, Pulmonary stenosis, abnl genetalia, retard of growth, deafness
PTPN11 sequencing (80%)
little boy with hundreds of freckles and hearing aid

fetal alcohol syndrome
physical and cognitive abnormalities in children caused by a pregnant woman’s heavy drinking

Fetal hydantoin syndrome
Also: dilantin embryopathy
Maternal use of anti-seizure drug phenytoin
Growth deficiencies, underdeveloped fingertoenails, mild dev delays, some cleft palate, small head, finger toe malformations

Infant of diabetic mother
Transposition of great vessels, Duodenal atresia, Anencephaly and neural tube defects

Acyanotic
absence of a bluish appearance of the skin and mucous membranes

VSD
Holosystolic, harsh-sounding murmur
Loudest at tricuspid area

megablastic anemia
due to thiamine or B12 deficiency
-decrease in purine and TMP = decrease in NA synthesis and affects cell division (marrow & gut)

Thalassemia
Inherited defect in ability to produce hemoglobin

Fanconi anemia
FANCA, B, C, D2, E, F, G Mostly Ar but some X-linked
short stature, abnl pigmentation, , AML, tumors of head and neck, radial polydactyly
FANCA seq and del/dup (66%)

Anaplastic
characterized by a loss of differentiation of cells and their orientation to one another; a characteristic of malignant tumors

Hereditary spherocytosis
RBC membrane defect in tethering proteins: spectrin/band 3.1/ankyrin,
Hemolytic anemia, jauncide, and splenomegaly often after URIs

Glucose-6-phosphate dehydrogenase deficiency
A hereditary condition in which red blood cells break down when the body is exposed to certain drugs.

Rome IV criteria
Constipation:
2 or less BMs / wk
Hx of stool withholding
Hx of pain/hard BMs
Large fecal mass in rectum
Hx of large diameter stools

eosinophilic esophagitis
a chronic immune system disease in which a type of white blood cell called an eosinophil builds up in the esophagus, usually as the result of an allergy to certain foods

vesicoureteral reflux
backflow of urine from the bladder into the ureters

Myoglobinuria
The abnormal presence of a hemoglobin-like chemical of muscle tissue in the urine; it is the result of muscle deterioration.

Which one best describes lesions associated with condyloma acuminatum?

a. Verruciform
b. Plaque-like
c. Vesicular
d. Bullous
a

39yo female has completed course of amox for strep throat. LMP was 2wks ago, says it was normal. On exam, there’s erythema of extern. genitalia w/small amount of white discharge. Micro wet prep reveals few clue cells, but many budding hyphae. No WBCs. Which one would be the most appropriate treatment?

a. Metronidazole 500mg BID x7 days
b. OTC hydrocortisone 1% cream TID
c. Fluconazole tabs 150mg x1 dose
d. Erythromycin 500mg TID x10 days
c

Woman c/o vaginal itching, white discharge. She is in good health except for recent abx for strep throat. Pelvic reveals tender vulvovaginal area w/edema and nonmalodorous white patches. Which is the most likely cause?

a. Bacterial vaginosis
b. Trichomonas
c. Lactobacillus overgrowth
d. Candidiasis
d

18yo female c/o secondary amenorrhea. On exam, there is normal secondary sex characteristics and normal genitalia. Pregnancy is ruled out. What would necessitate further eval?

a. Elevated blood cholesterol levels
b. Androgen deficiency
c. Galactorrhea
d. Hirsutism
c

24yo female is dx’d w/primary dysmenorrhea. Which med would be used as first-line to help control symptoms?

a. Antianxiety meds
b. Progesterone-only contraception
c. Oral steroids
d. NSAIDs
d

Primary amenorrhea is best described as:

a. Cessation of menstruation x6mo
b. Failure of menstruation to occur by 17ho
c. Failure of menstruation to occur by 13yo
d. Cessation of menstruation x6mo after menarche
c

25yo female c/o vaginal irritation and discharge. On exam, cervix is easily friable and erythematous. No adnexal tenderness. Wet prep reveals mobile protozoa on NS slide. This most likely represents:

a. Trichomonas
b. Mucopurulent cervicitis
c. Bacterial vaginosis
d. Gonorrhea
a

16yo female has h/o secondary amenorrhea. Menarche at 10yo, regular cycles x2yrs, has not menstruated x4yrs. What is most frequent etiology of this problem?

a. Eating disorder
b. Pregnancy
c. Anovulatory cycles
d. Stress
a

Woman is experiencing vaginal discharge. Wet mount with KOH would be used to confirm:

a. Herpes simplex
b. Gonorrhea
c. Candidiasis
d. Chlamydia
c

Treatment options for condyloma acuminatum include:

a. Imiquimod (Aldera)
b. Azithromycin
c. Acyclovir
d. Metronidazole
a

25yo postmenopausal female c/o pain in upper outer quadrant of L breast x1mo. Best course of action would be:

a. Reassure pt that pain is often not presenting symptom of breast cancer.
b. Teach pt breast self-exam.
c. Order labs as most likely this is secondary to hormonal fluctuation
d. Perform breast exam and order mammo
d

PID typically presents with all of the following except:

a. Dysuria
b. Leukopenia
c. Cervical motion tenderness
d. Abd pain
b

Which of the following are of a reproductive and pelvic origin?

a. Salpingo-oophoritis (fallopian tube/ovary) secondary to PID
b. Gynecologic malignancy
c. Adhesions
d. Myomata uteri
a

25yo female c/o tender area near her introitus and to the L of her perineum. Very painful sex was first sign. Initially bump was very small, but now is ping-pong ball size. On exam, abscess is present on L medial side of labia minora and there’s edema extending into perineum. What is dx?

a. Lipoma
b. Dermoid cyst
c. Bartholin’s cyst
d. Skene’s duct cyst
c

49yo female c/o dark, watery brown vaginal discharge. Which best describes what might be seen on physical exam in pt’s with cervical cancer?

a. Ulcerated firm cervix
b. Vague lower abd pain
c. Enlarged tender femoral lymph nodes
d. Soft, still shaped cervix
a

22yo female c/o pelvic pain. Exam reveals cervical motion and uterine tenderness. Which supports PID dx?

a. Temp <100F
b. Absence of WBCs in vag fluid
c. Mucopurulent vag discharge
d. Lab documentation of cervical infection w/E. coli
c

When educating pt about rationale for getting mammo, which statement is false?

a. Mammo is cost-effective method to screen for breast cancer
b. Mammo detects all breast cancers
c. Mammo should be accompanied by breast exam
d. Negative mammo should not delay biopsy of clinically suspicious mass
b

When educating women about breast cancer risk factors, which statement is incorrect?

a. Pregnancy after 35yo
b. Late menopause after 57yo
c. Fibrocystic breast dz
d. H/o maternal breast cancer
c

Which of the following statements is accurate regarding the usefulness of mammo in screening and detection of breast cancer?

a. Mammo shouldn’t be done if there is any breast pain or nipple retraction
b. All women >40yo should have mammo on annual basis
c. Mammo should be done annually for all women of child-bearing age
d. Mammos should be performed annually after initial pregnancy, especially if women doesn’t breastfeed
b

Which would be considered normal surface characteristic of the cervix during a speculum exam?

a. Small, yellow, raised around area on cervix
b. Friable, bleeding tissue opening of the cervical os
c. Red patch areas w/occasional white spots
d. Irregular, granular surface w/red patches
a

What is the most common cause of dysfunctional uterine bleeding?

a. Endocrine disorders
b. Stress
c. Anovulation
d. Anatomical abnormality
c

PMS occurs with greatest frequency and severity in the:

a. Late luteal phase
b. Midfollicular phase
c. Proliferative phase
d. Early luteal phase
a

Which is not a common cause of irregular menstrual bleeding?

a. Endocrine disorders
b. Stress
c. Anovulation
d. Anatomical abnormality
c

What is considered the primary etiology of primary dysmenorrhea?

a. Ovarian cysts
b. Prostaglandin production
c. Endometriosis
d. Adenomyosis
b

28yo female c/o breast tenderness, fatigue, abd bloating, fluid retention, irritability 1wk before her menses onset. What is most important info to obtain from this pt to determine if the pt has PMS?

a. Severity of symptoms
b. Occurrence of symptoms in menstrual cycle
c. Frequency and number of symptoms over past 4mo
b

35yo woman c/o 6mo h/o hypermenorrhea, backache, pelvic pressure. On exam, you discover 12wk size uterus w/irregular contour. What does this represent?

a. Uterine cancer
b. Dysfunctional uterine bleeding
c. Uterine fibroid
d. Fecal impaction
c

Female c/o vaginal itching and white discharge. Denies sexual activity or douching. In good health except for recurrent strep throat. Pelvic reveals tender vulvovag area w/edema and white patches. No odor. What is the most likely cause?

a. Bacterial vaginosis
b. DM
c. Allergy to personal hygiene product
d. Candidiasis after abx treatment
d

32yo woman c/o postcoital bleeding. Which would not be included in the initial assessment?

a. Pap smear
b. Uterine biopsy
c. Pelvic ultrasound
d. CBC w/diff
b

What phase of menstrual cycle begins with menses cessation and ends w/ovulation?

a. Ovulatory phase
b. Follicular phase
c. Proliferative phase
d. Luteal phase
b

What phase of menstrual cycle begins with ovulation and ends w/menstruation?

a. Ovulatory phase
b. Follicular phase
c. Proliferative phase
d. Luteal phase
c

Name 4 structural abnormalities that are causes of dysfunctional uterine bleeding.
PALM:
Polyps
Adenomyosis
Leiomyoma
Malignancy

Name 5 non-structural abnormalities that are causes of dysfunctional uterine bleeding.
COEIN:
Coagulopathy
Ovulatory disorders
Endometrial
Iatrogenic
Not classified

What is abnormal/dysfunctional uterine bleeding?
Acute or chronic bleeding from uterine corpus; abnormal in regularity, volume, frequency, or duration; occurs in pregnancy absence.

What is acute DUB?
Episode of sufficient quantity to require immediate intervention to prevent further blood loss

What is chronic DUB?
Present for the majority of the last 6mo

What is the most common benign tumor of the genital tract?
Leiomyomas? (Hollier CPG p. 772)

Name some risk factors for DUB.
Anovulation
Hormone replacement anovulation
Obesity
Nulliparity

35yo
DM
Personal/fam h/o coagulation disorder
Liver disorder
Anticoagulant therapy/chemo

What are some subjective findings for DUB?
Heavy bleeding
Bleeding >7 days
Cycles closer than 21 days
Pain
Post-coital bleeding
Passing clots/tissue
Dizziness
Hot flashes
Temp intolerance
Uterine/cervical tenderness

What are some objective findings for DUB?
Excessive bleeding on exam
Hypotension
Tachycardia
Diaphoresis
Vag atrophy
Mass
Trauma
Enlarged uterus/adnexa
Hirsutism
Thyromegaly
Bruising
Galactorrhea

What are some diff dx’s for DUB?
PALM-COEIN
Traumatic injury
Pregnancy-related bleeding (ectopic, SAB, placenta previa/abruptio)

What are some diagnostic lab studies for DUB?
-*Serum hCG: always do first before examining pt of menstruating age w/vag bleeding
-CBC
-ABO/Rh if preg. suspected/severe bleeding
-STD tests/wet prep
-Cervical cytology
-CMP (w/renal & liver panel)
-Coags
-If hx indicates: screen for bleeding disorders for DUB at menarche.
-If hx indicates: thyroid panel
-Endometrial bx

What are some diagnostic imaging studies for DUB?
-TVUS: first-line if imaging needed
-Saline infusion sonohysterography (helpful for leiomyoma)
-Hysteroscopy (can be used for guided endometrial bx)
-MRI

What are some diagnostic studies for polyps in DUB?
-TVUS
-Saline infusion sonography
-Hysteroscopy

What are some diagnostic studies for adenomyosis in DUB?
TVUS or MRI

What are some diagnostic studies for leiomyomas in DUB?
TVUS

What are some diagnostic studies for malignancy in DUB?
Bx/pathology

What are some diagnostic studies for coagulopathies in DUB?
Labs

What are some surgical options for DUB?
-Varies depending on dx, age, fertility status/desire
-If bleeding severe/pregn. related: D&C or hysterectomy in extreme cases
-Hysteroscopic removal of polyps
-Endometrial ablation
-Hysterectomy
-Traumatic repair

What are some pharm options for DUB?
-NSAIDs (cyclic DUB)
-Antifibrinolytics (cyclic DUB)
-Combined OCP, medroxyprogesterone acetate, levonorgestrel-releasing intrauterine systems (effective if agreeable to contraceptive effects)
-Danazol/GnRH agonists (reduce DUB if med/surg treatments fail/contraindicated
-Abx if infection/STD
-Intravag estrogen for vag atrophy

Which meds for DUB are contraindicated in pregnancy?
-Ethinyl Estradiol (Estinyl): estrogen; risk of VTE; use w/progesterone if pt hasn’t had hysterectomy.
-Medroxyprogesterone Acetate (Provera): progesterone; risk of VTE; caution w/CVA, CA risk
-Levonorgestrel-releasing intrauterine system (Mirena): combined hormone; risk of infection/migration; bleeding/cramping may occur x1-3mo
-Danazol (Danocrine): androgen; risk of acne/wt gain/hirsutism; short-term use for refractory DUB

Which meds for DUB are pregnancy cat C?
-ibuprofen (Motrin), naprosyn (Aleve): NSAID; risk of gastric upset/ulcer; don’t use w/other NSAIDs

Which med for DUB is preg cat B?
Tranexamic acid (Lysteda): antifibrinolytic; risk of VTE; don’t use w/hormones (risk of CVA/VTE)

What are some consultation/referral recommendations for DUB?
-Severity of acute DUB may warrant admission/consultation w/gyn
-Hematology: coagulopathies
-Endocrine: thyroid, hyperprolactinemia, PCOS findings
-Surg/Gyn: polyps, adenomyosis, leiomyoma

What are the follow up recommendations for DUB?
-Depends on type, severity, course of treatment
-Acute: 1-2wks to assess anemia prn
-Combined OCPs require at least 21-day course to eval efficacy, though improvement seen by ~80% in 3 days

What is the expected course of acute DUB?
Once bleeding stabilized: transition to maintenance therapy prn

What is expected course of chronic DUB?
Treatment should progress until DUB controlled; may required more testing, dosage adjustments, referral for treatment options

What are possible complications of DUB?
-Anemia
-Break through bleeding
-VTE
-Uterine artery embolization/polypectomy: maybe issues w/infertility
-Premature ovarian failure secondary to hysterectomy

What is the average age of menarche for Caucasians?
12.6y

What is average age of menarche for African-Americans?
12.1y

What is average age of menarche for Latinas?
12.2y

What does higher BMI in childhood result in regarding puberty?
Earlier onset

Timing and progression of puberty are related to…
Environmental factors, including:
-Socioeconomic conditions
-Nutrition
-Access to preventive healthcare

At what average age do secondary sexual development occur?
9y

Menarche usually occurs w/in __ after thelarche (breast buds).
2-3y

When should dx/referral be made for primary amenorrhea?
-Any adolescent w/out menarche by 15yo
-Any adolescent w/out menarche 3y after thelarche

What are some things that can cause menstrual irregularities?
-Disturbances in normal hormone release
-Significant wt loss
-Strenuous exercise
-Substantial changes in sleep/eating habits
-Severe stressors

Most cycles range from _.
21-34 days

When is woman’s normal cycle length usually established?
Around 6th gynecological yr, ~19-20yo

What are the 2 cycles that occur simultaneously in the menstrual cycle?
Ovarian and endometrial

What cycle are follicular, ovulation, & luteal phases?
Ovarian

What phase is menstrual days 1-14 in ovarian cycle?
Follicular

Prominent hormones: FSH, estrogen

Description: maturation of ovarian follicle

What phase is menstrual day 14 in ovarian cycle?
Ovulation

Prominent hormones: LH

Description: ovulation 36h after LH surge; increased basal body temp

What phase is menstrual days 15-28 in ovarian cycle?
Luteal

Prominent hormones: Progesterone, estrogen

Description: follicle becomes corpus luteum

What cycle are menses, proliferative, & secretory phases?
Endometrial/ovarian

What phase is menstrual days 1-5 (variable) in endometrial phase?
Menses (part of proliferative phase)

Prominent hormones: prostaglandin

Description: endometrium sloughs if fertilization of ovum doesn’t occur

What phase is menstrual days 1-14 in endometrial phase?
Proliferative

Prominent hormones: estrogen

Description: endometrium proliferates

What phase is menstrual days 14-28 in endometrial phase?
Secretory

Prominent hormones: progesterone

Description: endometrium thickens in prep for implantation

What are some common menstrual abnormalities?
Prolonged interval
Excessive flow

What are some menstrual conditions that may require eval?
-Absence of cycle 3y after thelarche
-Absence of cycle by 13yo w/out signs of pubertal development
-Absence of menses by 15yo
-New onset irregularity
-Cycles closer than 21 days or farther apart than 45 days
-Menses >7 days
-Menses needing pad/tampon change q1-2h
-Painful menses

What is a Bartholin’s gland cyst/abscess?
Obstruction of one or both Bartholin glands, resulting in pus that forms lump/abscess.

Where are Bartholin glands located?
Bilat to opening of vagina

What are typical characteristics of Bartholin’s gland cyst/abscess?
1-3cm, unilat

What is etiology for Bartholin’s gland cyst/abscess?
-Most common >60yo
-Mechanical irritation from tight fitting undergarments resulting in chronic inflammation
-STDs

What are some risk factors for Bartholin’s gland cyst/abscess?
-Vulvovaginal infection
-Poor perineal hygiene

What are some assessment findings of Bartholin’s gland cyst/abscess?
-Firm labia mass/cyst
-Erythema
-Induration
-Labia minora edema
-Low grade fever
-Possible purulent drainage

What are some diff dx for Bartholin’s gland cyst/abscess?
-Sebaceous cyst
-Malignancy/tumor

What are some diagnostic studies for Bartholin’s gland cyst/abscess?
-Usually none unless other infection suspected
-C/S of cyst contents
-Cx for STD, esp. gonorrhea & chlamydia

What is the most common causative organism of Bartholin’s gland cyst/abscess?
Aerobic bacteria E. coli

What are some preventive measures for Bartholin’s gland cyst/abscess?
-Loose breathable garments
-Good perineal hygiene
-Early treatment

What are some nonpharm treatment options for Bartholin’s gland cyst/abscess?
-None needed if only 1-2mm & asymptomatic
-Exclude possible carcinoma if >40yo
-Local moist heat
-Warm sitz baths/tepid water soak 3-4 times/day
-I&D w/Word cath of fluctuant abscess if refractory to initial treatment (no sex until cath removed)
-Possible marsupialization (permanent surgical opening allowing drainage)

What is the goal of treatment for Bartholin’s gland cyst/abscess?
To facilitate drainage of cyst contents

What are some pharm options for Bartholin’s gland cyst/abscess?
-If high risk/recurrent: Augmentin for E. coli/Strep, clinda for Staph (incl. MRSA)
-Treat for any STD
-NSAIDs

When is consultation/referral recommended for Bartholin’s gland cyst/abscess?
Surgical or gyn consult if large cyst or unresponsive to treatment

What are follow up recommendations for Bartholin’s gland cyst/abscess?
Re-eval in 7-10days

What is expected course of Bartholin’s gland cyst/abscess?
-Complete resolution w/appropriate treatment
-1 of 10 will recur

What are possible complications of Bartholin’s gland cyst/abscess?
Cellulitis

What is description of breast cancer?
-Malignant breast tumors
-Primarily female, but can be in males
-Stratified into 1 of 5 subtypes based on histopathological characteristics (Luminal A/B/B-like, HER2 positive, Triple Negative)

Presence of _____ and _________ gene mutations are associated with lifetime risk of breast ca from 45-60%.
BRCA 1, BRCA 2

What cancer is the most frequently diagnosed in females?
Breast

Which ethnic group is more likely to develop breast ca?
Caucasians

What are some risk factors for breast ca?
-Increasing age
-Dense breast tissue
-Prolonged estrogen exposure (menarche <12yo, 1st term pregn >35yo, nulliparity, never breastfeeding, contraceptives w/exogenous hormones, menopause >55yo, postmenopausal hormone therapy)
-Fam h/o breast ca (esp. 1st degree relative)
-Personal h/o breast ca
-H/o benign breast dz (nonproliferative = slight risk, proliferative = high risk)
-Obesity in postmenopause
-Inherited gene mutations
-High dose radiation exposure to chest area at <20yo ->3 ETOH drinks/day
-Emerging/unclear risk factors (tobacco, phys. inactivity, high fat diet, night shift)

What are some assessment findings in breast ca?
-Painless, firm, fixed mass (most common symptom; no changes in mass w/menstruation)
-Nipple discharge that’s not breast milk
-Skin/nipple changes (dimpling, skin ulceration, lymphedema, nipple retraction, scaly nipple lesion or eczematous rash i.e. Paget’s)
-Increased vascular pattern of breast
-Significant asymmetry of breasts
-Axillary, supraclavicular, infraclavicular lymph node enlargement
-Late findings: wt loss, anorexia, bone pain, anemia

What are some diff dx for breast ca?
-Breast cysts
-Fibroadenoma
-Sclerosing adenosis
-Intraductal papilloma
-Hyperplasia
-Radial scars
-Mastitis

What are some diagnostic studies for breast ca?
-Mammo
-US (differentiates fluid-filled cyst from solid mass)
-MRI
-Breast thermography (digital infrared thermal imaging)
-Fine needle or core bx
-Incisional or excisional bx

What are some preventions for breast ca?
-Screening (varies by expert organization)
-More frequent mammo or MRI for higher risk women
-Annual screening starting at 40yo (ACOG, ACOR, ACS, NCCN)
-Biennial screening 50-74yo (USPSTF)

If a woman has a breast mass but a normal mammo, what should be done?
NOT normal to have mass w/normal mammo; further imaging/eval needed.

What are some nonpharm options for breast ca?
-Genetic testing
-Tumor characteristic assessment
-Baseline bone scan
-CT abd/chest
-Breast conserving surgery (lumpectomy)
-Mastectomy (radical or prophylactic)
-Radiation
-Sentinel lymph node bx
-Axillary node dissection

What are some pharm options for breast ca?
Hormonal therapy:
-Tamoxifen
-Raloxifene
-Ca/Vit D suppl. recommended
-Anastrozole
-Letrozole

Chemo (adjuvant or neoadjuvant)

Radiation:
-Begin 4-6w after surgery or chemo
-Whole breast and accelerated partial breast.

When is consultation/referral recommended for breast ca?
All palpable masses to surgeon for eval & bx if indicated

What is recommended follow up for breast ca?
-H&P q3-6mo x3y after treatment, then q6-12mo x2y
-All women should do BSE
-Reg gyn follow-up (tamoxifen increases risk for endometrial ca; any vag bleeding should be eval’d)
-Mammo q6-12mo for surveillance, yearly once stability of mammo findings achieved
-MRI yearly if meets certain risk criteria

What is expected 5-year survival rate for noninvasive and Stage I breast ca?
Non (tumor <1cm w/no ax node involvement) & Stage I (tumor >1cm w/no ax node involvement) = 100%

What is expected 5-yr survival rate for Stage II breast ca?
93% (tumor <5cm w/ax node involvement)

What is expected 5-yr survival rate for Stage III breast ca?
72% (tumor >5cm w/chest wall or skin extension, inflammatory changes, or supraclavicular involvement)

What is expected 5-yr survival rate for Stage IV breast ca?
22% (metastatic)

What are some possible surgical complications of breast ca?
-Postop lymphedema
-Limited upper extrem. movement

What are some possible chemo complications of breast ca?
-N/v
-Alopecia
-Leukopenia
-Stomatitis
-Fatigue

What are some possible radiation complications of breast ca?
-Skin changes (burning, peeling, discoloration)
-Fatigue
-Axillary hair loss
-Rib fx, heart damage, radiation pneumonitis, brachial plexopathy, slight increased risk for secondary cancer (all rare)

What are some general possible complications of breast ca?
-Infertility
-Osteoporosis
-Musculoskeletal complaints
-Sexual dysf.
-Cog. changes
-Fatigue
-Depression
-Insomnia
-Neuropathy
-Cardiac toxicity
-CVD

What is fibroadenoma?
Benign breast lesion of fibrotic stroma (connective tissue) & glandular tissue; heterogenous lesion AKA benign breast dz (BBD)

What is etiology of fibroadenoma?
Unknown, thought to be exaggerated response to normal hormonal stimuli

What is the most common benign tumor in female breast?
Fibroadenoma (most common 15-40yo, but can be any age)

What are some risk factors for fibroadenoma?
-Multiple FA’s associated w/rare ca syndromes (Maffuci syndrome, Cowden syndrome, Carney complex)
-Increased risk of BBD with h/o adolescent ETOH consumption, esp. >/=10g per day

What are some assessment findings of BBD?
-Discrete, painless, firm/rubbery mass w/well-defined borders
-Freely mobile
-Fluctuations in size w/pregn. or menst. cycle
->5cm considered giant FA
-No nipple discharge

What are some diff dx for BBD?
-Fibrocystic breast dz
-Intraductal papilloma
-Breast ca
-Phyllodes tumor
-Cyst
-Other benign breast dz

What are some diagnostic studies for BBD?
-Mammo
-US
-Fine needle bx
-Open bx

What are some nonpharm options for BBD?
-Conservative observation, pt reassurance
-Surgical excision if >3cm, symptomatic, or if dx questionable due to imaging findings such as irreg. borders or abnormal vascularity.
-Cryoablation

What are some pharm options for BBD?
There are none

Does BBD affect lactation?
No

When is consult/referral recommended for BBD?
Surgical for excision of giant FA, symptomatic lesions, lesions w/questionable findings on imaging

What are follow-up rec’s for BBD?
Conservative management: US q6mo x2y after disovery

Postop as rec. by surgeon

What is expected course for BBD?
-May undergo spontaneous involution due to hormonal changes in younger/postmen.
-Subsequent lesions after involution require complete workup as new lesion.
-Complete resolution after surgical removal, but recurrence possible.

What are possible complications for BBD?
-Progression to malignancy if not excised
-Postop complications: infection, formation of scar tissue

What is fibrocystic breast dz?
-Spectrum of changes: cyst formation, columnar cell changes, mastalgia, apocrine metaplasia, sclerosing adenosis, blunt duct adenosis, epithelial hyperplasia, atypical ductal/lobular hyperplasia
-One of heterogenous lesions encompassed by term benign breast dz (BBD)

What are possible causes of FBD?
-Luteal phase defect
-Increased estrogen
-Hyperprolactinemia
-Hypersensitivity to estrogen
-Sensitivity to methylxanthines
-Dietary fat intake

What is typical age of FBD?
Premenop. 30-50yo

What are some risk factors for FBD?
-None definitively
-Possible methylxanthine ingestion (caffeine, chocolate, etc.)
-Increased risk in pt’s with h/o adolescent ETOH consumption (>/=10g/day)

What are assessment findings of FBD?
-Maybe asymptom.
-Palpation of smooth, movable masses, may vary in size
-Vague areas of breast thickening/ridges
-Breast pain/discomfort/tenderness, diminishes after menses
-Breasts feeling full, swollen, heavy, engorged
-Worsening of symptoms premenstr.
-Nipple discharge (varying color/consistency)

What are diff. dx for FBD?
-Breast ca
-Intraductal papillomas
-FA
-Mastitis
-Chest wall syndrome
-Neuralgia

What are diagnostic studies for FBD?
-Prolactin level
-TSH
-Mammo
-US
-Needle/open bx
-Maybe galactography (ductography) when nipple discharge present

What are some nonpharm options for FBD?
-Eval to rule out malign.
-Therapeutic aspiration of cysts
-Reassurance
-Cold compress
-Supportive bra worn 24h
-Na restriction 10d before menstr. onset
-Decreased/eliminated caffeine
-Reduce dietary fat
-Fine, core, excisional bx often needed to differentiate from malign.

What are some pharm options for FBD?
Spironolactone (Aldactone):
-For swelling/pain premenstr.
-Usually 100mg x5d, then retitrate (25-200mg/day)

Vit B6, E

Evening primrose oil:
-For mastalgia

OCPs

Danazol (Danocrine)

Bromocriptine (Parlodel):
-For more severe dz

Is there any affect on lactation with FBD?
No

When is consultation/referral recomm. for FBD?
Surgeon for eval of masses that are very painful/cause continuous pain, or any abnorm. on mammo/US

What is recomm. follow up for FBD?
Mammo as indicated by age/risk factors

What is expected course for FBD?
Benign but chronic condition

What are possible complications of FBD?
Maybe increased risk of malign. if atypical hyperplasia on bx, though >80% w/atyp. hyperplasia don’t develop invasive ca in their life.

What is intraductal papilloma?
-Benign tumor in ductal system of breast
-Part of BBD group

What is cause of IP?
Unknown, but maybe proliferation of ductal epithelium

What is usual age for IP?
30-50yo

What are risk factors for IP?
-Mult. papillomas increase lifetime risk of breast ca
-Increased risk of BBD in adolescent ETOH >/=10g/day

What are assessment findings for IP?
-Smooth, small mass (may/may not be palp)
-Typically retroareolar
-Bloody/serous nipple discharge
-Usually unilat unless mult. ducts involved

What are some diff dx for IP?
-Breast ca
-Galactorrhea
-In situ carcinoma
-Ductal ectasia
-Epithelial hyperplasia
-Physiological or gestational nipple discharge

What are some diagnostic studies for IP?
-Mammo
-US (more sensitive than mammo)
-MRI
-Galactography (ductography)
-Fine needle/core bx

What are nonpharm options for IP?
Surgical excision (microdochectomy)

What are pharm options for IP?
None

Does surgical excision for IP affect breastfeeding?
Not unless major/total duct excision required

When is consult/referral recomm. for IP?
Surgical for excision

What are follow up recomm. for IP?
Postop as recomm. by surgeon

What is expected course for IP?
Complete resolution after excision w/out recurrence

What are possible complications of IP?
-Potential for atypical/mult. lesions to progress to malig. if not excised
-Postop comp.: infection, hematoma, scar tissue

What is cervical ca?
Cervical epithelial changes (precancerous phase, usually in squamocolumnar junction affecting squamous cell epithelia = 80-90%; or columnar epithelia = 10-12%) that progresses to malig. (can be invasive)

What is etiology of cervical ca?
-Oncogenic strains of HPV
->200 strains of HPV ID’d: high risk or low risk
-High risk: strains 16 & 18 responsible for 70% of all cerv. ca; produce viral proteins that disable body’s tumor suppression genes –> increased proliferation of abnorm. cells

What is 3rd most common genital ca in U.S.?
Cervical ca

What ethnicities have highest rates of cervical ca?
Hispanic & Caucasian

What is common age for cervical ca?

30yo

What are risk factors for cervical ca?
-First intercourse at 18yo or younger
-HPV infection (current or previous)
-HIV, AIDS, other immunocomp. state
-Cigarette smokers (2-3x higher incidence)
-Mult. sex partners
-Diethylstilbestrol (DES) exposure in utero (incr. risk of clear cell carcinoma)
-Long-term OCP (up to 3-4x)
-3 or more births
-Fam h/o cervical ca
-HSV-2, chlamydia

What are assessment findings in cervical ca?
Subj:
-Often asympt. in early stages w/variable or non-spec. symptoms
-Abnorm. uterine bleeding (in 80-90% of pts)
-Watery/purulent vag discharge
-Dyspareunia
-Hematuria
-Bladder outlet obstruction
-Pelvic/back pain (invasion of tumor)
-Constipation
-Leg swelling from lymphatic/vascular obstruction

Obj:
-Cervix may be normal in early stages
-Cervical erosion, ulcerations, bleeding mass
-Decr. mobility/hardening of cervix
-Irreg., cauliflower-like growth
-Rectal exam: bleeding from tumor compression
-Fistula

What are diff. dx for cervical ca?
-Severe cervicitis
-Cervical polyp
-Endometrial carcinoma
-Vaginitis
-PID

What are some diagn. studies for cervical ca?
-Pap
-Colposcopy w/endocervical bx, LEEP, and/or cold knife cone bx
-Cystoscopy/proctoscopy for advanced/large tumors
-Staging based on International FIGO system
-CBC, CMP, renal/hepatic function tests
-MRI, CT, CXR, PET

What are some preventions for cervical ca?
-Have 1st Pap at 21yo
-21-29yo: Pap q3y unless risk factors ID’d (then more frequ.)
-30-65yo: Pap/co-test for HPV q5y, or Pap alone q3y
-Avoid HPV infection via abstinence or barriers
-HPV vax: 9-26yo (prevents most common strains, genital warts)
-Smoking cessation

What are some nonpharm options for cervical ca?
Treatment based on staging

Surg:
-Cryosurgery, laser ablation, LEEP (early stages)
-Total/radical hyster.
-Cone bx
-Lymph node dissection
-Radical vag trachelectomy (RVT) w/lap pelvic lymphadenectomy (fertility-preserving method)
-Pelvic exenteration

Radiation:
-External beam/brachytherapy

What are some pharm options for cervical ca?
Chemo (adjuvant to other therapies):
-Cisplatin (Platinol): n/v, ototox, nephrotox, myelosuppresion; pretreatment hydration essential, monitor for extravasation.
-Ifosfamide (Ifex): hem. cystitis, myelosupp; concomitant infusion w/Mesna to prevent hem. cystitis; monitor PO/IV hydration >2L/day.
-Fluorouracil (Adrucil): n/v/d, hand-foot syndrome, mucositis, myelosupp; hospitalize at initiation due to risk of severe toxic reaction.
-Paclitaxel (Taxol): myelosupp., alopecia, n/v/d, mucositis, renal impairment; pretreat w/corticoster., benadryl, H2 antagonist.
-Topotecan (Hycamtim): myelosupp., n/v, alopecia, fever, dyspnea, rash, cough; get baseline neutrophil/plt counts, continue to monitor for bone marrow suppression.
-Bevacizumab (Avastin): weakness, HTN, abd pain, vomiting, GI bleed; impairs wound healing, d/c before surgery

What are some preg/lact. considerations in cervical ca?
-If found in early stages: pregn. may be allowed to continue w/surgical intervention several wks after deliv.
-If found later, preg. term. or immediate deliv. must be considered.

What are consult/referral recomm. for cervical ca?
-Surgical/onc
-Gyn/onc
-Radiation/onc
-Radiology
-Psychologist, social work, rehab specialists

What are follow up recomm. for cervical ca?
H&P w/spec exam, bimanual, pelvic, rectal, and cytology:
-Q3mo x2y
-Q6mo x3-5y
-Annually after 5y

What is expected course of cervical ca?
5y survival rate:
-Stage 0 (carcinoma in situ): 93%
-Stage IA (earliest stage 1): 93%
-Stage IB (can be seen w/out microscope): 80%
-Stage IIA (invasion beyond uterus, but no parametrial invasion): 63%
-Stage IIB (w/obvious parametrial invasion): 58%
-Stage IIIA (invasion of lower 1/3 of vag, no invasion of pelvic wall): 35%
-Stage IIIB (invasion of pelvic wall): 32%
-Stage IVA (invasion of bladder, rectum, beyond true pelvis, w/spread to adjacent organs): 16%
-Stage IVB (spread to distant organs): 15%

What are possible complications of cervical ca?
-Malig. recurrence
-Cystitis, proctitis
-Intestinal fistula
-Rectovag fistula
-SBO
-Vag stenosis
-Neuropathy
-Secondary malig.
-Emotional impact

What are most complications of cervical ca related to?
Specific therapy or treatment

What are some estrogen-containing combined contraceptives?
-Combined PO
-Contraceptive patch
-Vag ring

What are some progestin-only contraceptives?
-Progestin-only PO (Mini-Pill or POP)
-Medroxyprogesterone acetate (Depo, DMPA)
-Etonogestrel implant (Nexplanon)
-Emergency contraception (levonogestrel x1 dose aka Plan B, Ulipristal Acetate x1 dose Rx only aka Ella, copper IUD)

What are some long-acting reversible contraceptives (LARC)?
-IUD/IUS (ParaGard Copper T380A, Mirena, Skyla)

What are some barrier methods of contraception?
-Diaphragm
-Vag sponge
-Male condom
-Female condom
-Spermicide

What are some forms of sterilization?
-Tubal ligation
-Vasectomy
-Transcervical sterilization

What is MOA of combined-oral contraceptives?
-Suppress pituitary gonadotropins (FSH, LH) inhibiting ovulation
-Phasic pills to mimic hormonal fluctuations in menst. cycle

What are the 4 types of combo oral contraceptives?
-Monophasic
-Biphasic
-Triphasic
-Quadriphasic

How do monophasic COCs work?
Estrogen & Progestin doses fixed throughout cycle

How do biphasic COCs work?
-Estrogen dose remains same for 1st 21d of cycle
-Progestin dose is lower in 1st half of cycle
-Progestin dose is higher in 2nd half

How do triphasic COCs work?
-Estr. dose may be same or may vary throughout cycle
-Progest. dose varies throughout

How do quadriphasic COCs work?
Estr. & progest. doses vary throughout cycle

What are some SEs of too much estrogen in OCPs?
-Nausea
-Bloating
-HTN
-Breast tenderness
-Edema

What are some SEs of too little estrogen in OCPs?
-Early or midcycle breakthrough bleeding
-Incr. spotting

What are some SEs of too much progestin in OCPs?
-Incr. appetite
-Wt gain
-Fatigue
-Mood changes

What are some SEs of too little progestin in OCPs?
-Late breakthrough bleeding
-Amenorrhea

What is the theoretical effectiveness of COCs?
99.7%

What is the actual effectiveness of COCs?
91%

What are advantages of COCs?
-Very reliable temporary method when used properly
-No sexual activity interference

Non-contraceptive related benefits:
-Decr. menstr. flow & s/s of dysmenorrhea
-Acne improvement
-Menses regularity
-Anemia protection
-Menstr. s/s improvement
-Reduced ovarian cysts

What are disadvantages of COCs?
May cause life-threatening/serious complications:
-Thrombophlebitis/embolism
-Hepatocellular adenomas
-CVA
-Gallbladder dz
-HTN

Must remember to take pill every day

Provide no protection against STDs

May decr. milk prod. in lact. women

What are some general SEs of COCs?
-Nausea
-Breast fullness/tenderness
-Cyclic wt gain/fluid retention
-Breakthrough bleeding (esp. in 1st 3mo)
-Decr. menstr. flow/amenorrhea
-Fatigue
-Acne
-Mild HA
-Incr. appetite

What are some absolute contraindications for COCs?
-<6wks postpartum -H/o or current thrombophleb./thromboemb. disorder (incl PE, thrombogenic mutations) -BP >160/100
-Mult. risk factors for CAD
-Complex valvular heart dz
-Migraine w/aura
-DM w/end-organ damage
-Known breast ca
-Active liver dz/liver ca
-Pregn (known/susp.)
-Undiagnosed gyn bleeding
-Uncontrolled HTN
-Major surg. w/prolonged immobilization

What are some relative complications for COCs?
->35yo & smokes <15 cig/day -Breastfeeding >/= 6wks but <6mo -Postpartum <3w -Migraines that start after initiating OCPs -SBP 140-159 or DBP 90-99 -Well controlled HTN -Hyperlipidemia -H/o breast ca, but no active dz x5y -Active gallbladder dz -DM -Sickle cell ->50yo
-Conditions that make it difficult to take pills consistently/correctly

What diagnostic studies should be done before initiating COCs?
-Consider Pap
-UPT
-STD screening as indicated

What are the danger signs of OCPs, contr. vag. ring, & contracept. patch?
ACHES:
-A: severe abd pain (maybe hepatic tumors)
-C: severe CP/SOB
-H: severe HA
-E: eye probs (blurry, flashing lights, blindness)
-S: severe leg pain

What is some pt education for COCs?
-No STD protection
-Smoking cessation
-Maintain ideal body wt
-Exercise
-Use back-up if on Rifampin (significant effect), barbiturates, Dilantin, phenylbutazone, griseofulvin, ampicillin, tetracycline, Tegretol.
-Take pills at same time every day

Missed pills:
-1 pill: take ASAP, unless not remembered until time for next pill, then take 2
-2 consec. pills: take 2 per day x2d, then resume 1/day. Use vag spermicide & condoms for remainder of cycle.
-3 or more: depends on what type of COC and where in the pack it was missed. Refer to product info regarding specific pill.

Breakthrough bleeding:
-50% have BTB in 1st 3mo, usually abates
-Does not indicate decreased effectiveness
-Usually caused by missing pills

What are follow up recomm. for COCs, contr. vag. ring, & contracept. patch?
-Eval 3mo after starting pills, then annually
-Check wt/BP after 3mo on pills
-Breast exam/Pap as indicated

How is transdermal contraceptive patch used?
-Apply on 1st Sunday after menses
-Wear x1wk
-Replace x3 consec. weeks
-No patch on 4th week, resulting in w/drawal bleed

What is MOA of contracept. patch?
Suppresses pituitary gonadotropins (FSH, LH), inhibiting ovulation

What is theoretical effectiveness of contracept. patch?
99.7%

What is actual effectiveness of contracept. patch?
91%

What are advantages of contracept. patch?
-Weekly application less arduous than daily pill
-Very reliable temp. method when used properly
-No interference w/sex

Non-contracept. benefits:
-Decr. menstr. flow/symptoms of dysmenorrhea
-Acne improvement
-Menses regularity
-Anemia protection
-Menstr. symptom improvement
-Reduced ovarian cysts

What are disadvantages of contracept. patch?
May cause life-threatening/serious complications:
-Thrombophlebitis/embolism
-Hepatocellular adenomas
-CVA
-Gallbladder dz
-HTN

Must remember to change patch

Provide no protection against STDs

May decr. milk prod. in lact. women

What are some SEs of contracept. patch?
-Nausea
-Breast fullness/tenderness
-Cyclic wt gain/fluid retention
-Breakthrough bleeding (esp. in 1st 3mo)
-Decr. menstr. flow/amenorrhea
-Fatigue
-Acne
-Mild HA
-Incr. appetite
-Skin irritation where patch applied

What are some absolute contraindications of contracept. patch?
-<6wks postpartum -H/o or current thrombophleb./thromboemb. disorder (incl PE, thrombogenic mutations) -BP >160/100
-Mult. risk factors for CAD
-Complex valvular heart dz
-Migraine w/aura
-DM w/end-organ damage
-Known breast ca
-Active liver dz/liver ca
-Pregn (known/susp.)
-Undiagnosed gyn bleeding
-Uncontrolled HTN
-Major surg. w/prolonged immobilization

What are some relative contraindications of contracept. patch?
-May be less effect. if wt >=198lbs
->35yo & smokes <15 cig/day -Breastfeeding >/= 6wks but <6mo -Postpartum <3w -Migraines that start after initiating OCPs -SBP 140-159 or DBP 90-99 -Well controlled HTN -Hyperlipidemia -H/o breast ca, but no active dz x5y -Active gallbladder dz -DM -Sickle cell ->50yo
-Conditions that make it difficult to change patch consistently/correctly

What is some pt education for contracept. patch?
-Apply to upper arm, back, abd, or butt (NO breasts)
-If partially detaches/completely removed, reapply/tape on as long as it has been off for <24h
-No STD prevention

How is contracept. vag ring used?
-Insert into vag 1st Sunday after menses
-Wear x1wk
-Replace x3wks
-No ring on 4th wk (results in w/drawal bleed)
-Can be used off-label: wear x4wks, then replace.

What is MOA of contracept. vag ring?
Suppresses pituitary gonadotropins (FSH, LH) inhibiting ovulation

What is theoretical effectiveness of contr. vag. ring?
99.7%

What is actual effectiveness of contr. vag. ring?
91%

What are advantages of contr. vag. ring?
-Monthly or weekly insertion may be less arduous than daily med
-Very reliable temp. method when used properly
-No interference w/sex

Non-contracept. benefits:
-Decr. menstr. flow/symptoms of dysmenorrhea
-Acne improvement
-Menses regularity
-Anemia protection
-Menstr. symptom improvement
-Reduced ovarian cysts

What are disadvantages of contr. vag. ring?
-Some pts may be uncomfortable inserting

May cause life-threatening/serious complications:
-Thrombophlebitis/embolism
-Hepatocellular adenomas
-CVA
-Gallbladder dz
-HTN

Must remember to change ring

Provide no protection against STDs

May decr. milk prod. in lact. women

What are some SEs of contr. vag. ring?
-Nausea
-Breast fullness/tenderness
-Cyclic wt gain/fluid retention
-Breakthrough bleeding (esp. in 1st 3mo)
-Decr. menstr. flow/amenorrhea
-Fatigue
-Acne
-Mild HA
-Incr. appetite
-Skin irritation/vaginitis

What are absolute contraindications for contr. vag. ring?
-<6wks postpartum -H/o or current thrombophleb./thromboemb. disorder (incl PE, thrombogenic mutations) -BP >160/100
-Mult. risk factors for CAD
-Complex valvular heart dz
-Migraine w/aura
-DM w/end-organ damage
-Known breast ca
-Active liver dz/liver ca
-Pregn (known/susp.)
-Undiagnosed gyn bleeding
-Uncontrolled HTN
-Major surg. w/prolonged immobilization

What are relative contraind. for contr. vag. ring?
->35yo & smokes <15 cig/day -Breastfeeding >/= 6wks but <6mo -Postpartum <3w -Migraines that start after initiating OCPs -SBP 140-159 or DBP 90-99 -Well controlled HTN -Hyperlipidemia -H/o breast ca, but no active dz x5y -Active gallbladder dz -DM -Sickle cell ->50yo
-Conditions that make it difficult to change ring consistently/correctly

What is some pt education for contr. vag. ring?
-Ring can lie anywhere w/in vag, but may be more comfy in deep/post. portion
-Ring can be left in at all times, incl. intercourse
-If ring falls out or is removed during intercourse, rinse off and reinsert w/in 3h
-If ring is out for <3h, no back-up necessary -If ring is out for >3h, back-up required x7d
-No STD prevention
-Store ring 68-77F out of direct sunlight

What is MOA for progestin-only pills (Mini-pill or POP)?
-Suppresses ovulation
-Creates thin, atrophic endometrium
-Thickens cervical mucous, making sperm penetration difficult

What is theoretical effectiveness for POPs?
99.7% (nearly 100% in lactating women)

What is actual effectiveness for POPs?
91%

What age is highest failure rate for POPs?
<40yo

What are advantages of POPs?
-Doesn’t alter quality/quantity of breast milk
-Can be used w/CVD risk factors
-Similar to OCPs: protects against develop. endometrial ca, decr. risk of PID
-Decr. menstr. cramps
-Eventually less heavy bleeding, shorter menses
-Decr. PMS symptoms
-Decr. breast tenderness

What are disadvantages of POPs?
-Must be taken every day at same time
-Missing only 1 pill will substantially incr. risk of preg.
-Menstr. cycle changes, esp. initially
-Incr. (rare) chance of ectopic

What are SEs of POPs?
-Nothing serious
-Menstr. cycle changes: spotting, BTB, prolonged cycles, eventually amenorrhea
-Breast tenderness
-HA
-Mood changes

What are indications for POPs?
-Unacceptable estrogen-related SE (GI upset, breast tenderness, decr. libido)
-Severe HA or HTN while on OCPs
-Absolute or relative contraind. to estrogen & COCs

What are absolute contraind. for POPs?
-Known/susp. breast ca
-Preg (known/susp)
-Medical condition worsened by fluid retention (CHF, mitral stenosis, pulm HTN)
-Undiagnosed gyn bleeding
-Liver dz

What are relative contraind. for POPs?
-Active thrombophleb./thromboemb. disorder
-H/o MI, ischemic heart dz, CAD
-Rifampin, barbiturates, phenytoin, carbamazepine, phenylbutazone

What are diagnostic studies before prescribing POPs?
-Pap as indicated
-STD screening as indicated

What is pt education for POPs?
-Most will experience irreg. menstr. bleeding w/spotting, BTB, prolonged cycles, amenorrhea
-If no h/o unprotected intercourse/not currently taking OCPs, start mini-pills immediately
-If changing from COCs to POPs, start on 1st day of menses or anytime during COC cycle
-Use backup x1mo after starting pills
-If 1 pill missed (>3h), take ASAP and use backup for remainder of pack

What are follow up recomm. for POPs?
-Eval 3mo after starting, then annually
-Notify HCP when d/c’ing breastfeeding

How is medroxyprogesterone acetate (Depo) used?
-IM (slowly released/provides contraception x13wks)
-150mg IM or 104mg SQ
-SQ easier/less painful, could have fewer SE (less bone density loss)
-Ideally given w/in 7d of menses onset or w/in 5d of giving birth

What is MOA of DMPA/Depo?
-Suppresses ovulation
-Creates thin, atrophic endometrium
-Thickens cervical mucous making sperm penetration difficult

What is theoretical effectiveness of DMPA/Depo?
99.8%

What is actual effectiveness of DMPA/Depo?
94%

What are advantages of DMPA/Depo?
-Doesn’t alter quality/quantity of breast milk
-Can be used w/CVD risk factors
-Similar to OCPs: protects against develop. endometrial ca, decr. risk of PID
-Decr. menstr. cramps
-Eventually less heavy bleeding, shorter menses
-Decr. PMS symptoms
-Decr. breast tenderness
-13wk period of effectiveness

What are disadvantages of DMPA/Depo?
-May lose significant bone mineral density (greater w/longer use, may not be reversible
-Only use long-term (>2y) if other BC methods are inadequate
-Fertility may not return for up to 2y after last injection (most will be fertile w/in 1y)
-Requires injection

What are SEs of DMPA/Depo?
-Amenorrhea
-Irreg. spotting/bleeding
-Heavy vag bleeding
-HA
-Wt changes esp. gain
-Libido changes
-Depression
-Dizziness

What are contraind. for DMPA/Depo?
-<6wks postpartum -Active thrombophleb. -BP >=160/100
-CAD
-Known breast ca
-Active liver dz
-Pregn (known/susp.)
-Undiagnosed gyn bleeding
-Known sensitivity to medroxyprogesterone

What diagnostic studies should be done before administering DMPA/Depo?
-Pap as indicated
-STD screen as indicated
-UPT

What is pt education for DMPA/Depo?
-SE profile
-Menses may not return x3-12mo after last injection
-No STD protection

What is recomm. follow up for DMPA/Depo?
-Repeat inj. q13wks
-Annual exam
-Consider bone density scan/alternative method after 2y
-Consider concurrent use of Ca/Vit D
-Hct/Hgb if h/o heavy/persistent menstr. bleed
-Preg. test if no reg. menses and/or s/s of preg.

How is etonogestrel implant (Nexplanon) used?
-Thin, plastic rod placed SQ in upper arm
-Releases hormone daily x3y

What is MOA of Nexplanon?
-Suppresses ovulation
-Creates thin, atrophic endometrium
-Thickens cervical mucous making sperm penetration difficult

What is effectiveness of Nexplanon?
99.9%

What are advantages of Nexplanon?
-After insertion, no additional attention required
-Doesn’t interfere w/sex
-Safe for all body wts
-No effect on milk prod.
-Effective x3y
-Quick return of fert.

What are disadvantages of Nexplanon?
-Requires office procedure for insertion
-SE may require removal

What are SE of Nexplanon?
-Amenorrhea
-Irreg. spotting/bleeding
-Heavy bleeding
-Wt changes (gain)
-HA
-Libido loss
-Vag dryness

What are contraind. for Nexplanon?
-Current/past h/o thrombosis/thromboemb. disorders
-Known/susp. preg.
-Liver tumors (benign or malig.), active liver dz
-Undiagnosed abnorm. vag bleeding
-Known/susp. breast ca, personal h/o breast ca (or other progestin-sensitive ca), now or h/o
-Allergic reaction to any rod components

What are diagnostic studies to be done before implanting Nexplanon?
-Pap if indicated
-STD screen if indicated
-Preg. test

What is Pt education for Nexplanon?
-SE
-Give pt info card that comes w/device
-If inserted w/in 5d of menses, no backup required
-No STD prevention

What are follow up recomm. for Nexplanon?
-Annual exam
-Remove device if become preg.

What is effectiveness of emerg. contracept.?
Estimated 85% of women who would have otherwise become preg.

What is MOA of emerg. contracept.?
-Inhibits/delays ovulation
-May inhibit fertilization/implantation of fertilized egg

What are advant. of emerg. contracept.?
-Backup for barrier method failure, missed OCPs
-Plan B One Step available OTC
-May use more than once in cycle
-No reports of teratogenic effects when fails

What are disadv. of emerg. contracept.?
-Limited window for use
-Ulipristal acetate requires Rx
-Copper IUD requires office visit to insert

What are SE of emerg. contr.?
-N/V
-Spotting
-HA
-Breast tenderness

What are indications for emerg. contr.?
-Unprotected intercourse w/in past 72-120h (method-dependent), preg. not desired

What are absolute contraind. for emerg. contr.?
-<6wks postpartum -H/o or current thrombophleb./thromboemb. disorder (incl PE, thrombogenic mutations) -BP >160/100
-Mult. risk factors for CAD
-Complex valvular heart dz
-Migraine w/aura
-DM w/end-organ damage
-Known breast ca
-Active liver dz/liver ca
-Pregn (known/susp.)
-Undiagnosed gyn bleeding
-Uncontrolled HTN
-Major surg. w/prolonged immobilization

What are relative contraind. for emerg. contr.?
->35yo & smokes <15 cig/day -Breastfeeding >/= 6wks but <6mo -Postpartum <3w -Migraines that start after initiating OCPs -SBP 140-159 or DBP 90-99 -Well controlled HTN -Hyperlipidemia -H/o breast ca, but no active dz x5y -Active gallbladder dz -DM -Sickle cell ->50yo
-Conditions that make it difficult to take pills consistently/correctly
-Preg.
-<4wks postpartum
-H/o STD in past 3mo
-Immediate post-septic abortion
-Unexplained vag. bleed
-Cervical, ovarian, endometrial ca
-Uterine fibroids that distort uterine cavity
-Anatomical abnorm. that make insertion difficult
-Current infection of cervix, uterus, fallopian tubes, postpartal endometritis, or infected abortion

Copper:
-Wilson’s dz
-Allergy to copper

Progesterone:
-Current/past breast ca
-Breastfeeding
-Acute DVT

What is pt education for emerg. contrac.?
Hormonal:
-May have spotting after taking
-No STD protection
-IDDM may need to monitor BGL closely

IUD:
-Barrier method until re-eval after next menses
-Monthly string checks after menses (notify if can’t locate)
-Change ParaGard q10y
-S/S of infection: abd pain, fever/chills, unusual bleeding, foul smelling vag discharge, fever w/vag discharge, string disappearance, painful intercourse

What are follow up recomm. for emerg. contrac.?
-If no menstr. bleeding w/in 3wks of emerg. contrac.
-If ments. bleeding <2d in duration
-If develop. of early preg. signs

How are IUDs/IUSs used?
Device placed/retained in uterine cavity

What is MOA of IUDs?
Unknown

Hypotheses:
-Incre. motility in fallopian tubes, moving ovum through rapidly
-Inflamm. effects on endometrium
-Local FB inflamm. response interfering w/sperm survival, motility, and/or capacitation
-Copper in ParaGard interferes w/estrogen uptake, potentiates local inflamm. response, may decr. sperm motility
-Levonorgestrel in Mirena/Skyla thickens cervical mucous, thins endometrium

What is effectiveness of IUDs?
-Copper: 99.2%
-Mirena: 99.8%
-Skyla: 99.1%

What are advant. of IUDs?
-Undetectable during intercourse
-No systemic effect on hormones
-Long-term contracep.
-Cost-effective if used long-term
-Good choice for lactation (no effect on milk prod.)
-Progest. IUSs decr. bleeding

What are disadv. of IUDs?
-2-5% of pts spontaneously expel IUD
-No STD protection

What are SE of IUDs?
-Abd infection/adhesions
-Sepsis
-Cervical infection/erosion
-Ovarian cysts
-Ectopic
-Embedment of IUD in uterus
-Infertility
-Spotting between periods
-Dysmenorrhea
-Preg.
-Prolonged/heavy menstr. flow (esp. copper)
-Amenorrhea (esp. progest.)

What are indications for IUD?
-Desire for long-term contracept.
-May be considered for adolescents (esp. Skyla because it’s smaller)
-May be safely used in parous and nulliparous

What are contraind. for IUD?
-Preg.
-<4wks postpartum
-H/o STD in past 3mo
-Immediate post-septic abortion
-Unexplained vag. bleed
-Cervical, ovarian, endometrial ca
-Uterine fibroids that distort uterine cavity
-Anatomical abnorm. that make insertion difficult
-Current infection of cervix, uterus, fallopian tubes, postpartal endometritis, or infected abortion

Copper:
-Wilson’s dz
-Allergy to copper

Progesterone:
-Current/past breast ca
-Breastfeeding
-Acute DVT

What are diagnostic studies to do before IUD insertion?
-STD risk assessment
-Preg. test

What are SE of LARC (IUDs)?
-Barrier method until re-eval after next menses
-Monthly string checks after menses (notify if can’t locate)
-Change ParaGard q10y, Mirena q5y, Skyla q3y
-S/S of infection: abd pain, fever/chills, unusual bleeding, foul smelling vag discharge, fever w/vag discharge, string disappearance, painful intercourse

What are follow up recomm. for IUDs?
-Eval after next menses, then annually
-Preg. test if amenorrheic

How does the diaphragm work?
-Dome-shaped latex cup w/flexible spring rim filled w/spermicide
-Placed in upper vagina to cover cervix completely
-Requires training/experience to fit correctly

What is MOA of diaphragm?
-Latex dome forms barrier between cervix & semen, preventing sperm from entering uterus
-Spermicide is used w/diaphragm for additional protection, killing any sperm that accidentally slip past rim

What is theoretical effectiveness of diaphragm?
94%

What is actual effectiveness of diaphragm?
88%

What are advant. of diaphragm?
-Helps prevent STDs
-No serious SE
-Insertion may be incorporated into foreplay
-Decr. incidence of cervical neoplasia
-No impact on future fertility

What are disadvant. of diaphragm?
-Not as effective as hormonal methods
-Must be refitted in wt change, after delivery/cervical surgery
-Must be used each time intercourse occurs
-Spermicide must be used w/device, more added for each additional intercourse event
-Must be left in x6h after sex
-Decr. effectiveness w/incr. frequ. of sex
-Requires dexterity to insert properly
-May be embarrassing or considered messy for women who dislike touching their genitals

What are SE of diaphragm?
-May be uncomfortable
-Some have recurrent UTIs
-Possible allergic reaction to latex/spermicide
-Bact. infections
-Vag. yeast infection
-TSS (rare)
-Vag trauma/ulceration caused by excessive rim pressure/prolonged wear

What are contraind. for diaphragms?
-Latex allergy
-Spermicide allergy
-H/o TSS
-H/o frequent UTI
-Abn. of uterine anatomy preventing satisfactory fit (uterine prolapse, extreme uterine retroversion, vag septum, severe cystocele/rectocele)
-Inability of pt/partner to learn correct insertion
-Full-term preg. delivered in past 6wks

What is dysmenorrhea?
-Painful cramping a/w menstruation
-Sweating
-HA
-N/v/d

What is etiology of primary dysmen.?
-No underlying pathology
-Incr. prostagland. cause plt aggregation, vasoconstric., uterine contract. which incr. likelihood of uterine ischemia

What is etiology of secondary dysmen.?
-Congenital anom. of uterus/vag
-PID, STDs
-Adenomyosis
-Endometriosis
-Pelvic tumors
-IUD
-Sexual trauma

What is predominant age for prim. dysmen.?
Teens to early 20’s

What is predominant age for second. dysmen.?
20’s – 30’s

What are risk factors for dysmen.?
Primary:
-Nulliparity
-Pos. fam. hx
-Cig. smoking
-Heavy menses
-Stress

Secondary:
-Pelvic infection
-STD
-Endometriosis

Dysmen. assessment findings:
-Pelvic cramping
-Heavy sensation in pelvis
-Malaise
-HA
-N/v/d
-Back/thigh pain
-Urinary frequ.
-Fatigue

Primary:
-Occurs usually in 1st 2 days of menstr.
-Located in suprapubic area, radiates to back/thighs
-May be a/w n/v/d

Secondary:
-Pain may refer to area of underlying pathology

Diff. dx for dysmen.:
-Intrauterine polyps
-IUD
-Pelvic/genital infection
-Endometriosis
-Uterine/ovarian tumors
-Adhesions
-Incompl. abortion
-Ectopic
-Preg. complications
-Ovarian cysts
-Polyps
-Interstitial cystitis
-Chronic pelvic pain
-UTI
-Adhesions

Diagnostic studies for dysmen.:
Primary:
-Usually none indicated
-Laparoscopy if unable to diagnose based on H&P

Secondary:
-WBC (elev. in infections)
-Cervical cx
-TVUS
-UA
-hCG

Nonpharm treatment for dysmen.
Primary:
-Heating pad
-Hot bath
-Exercise
-Pelvic exercise
-Relaxation
-TENS
-Laparoscopic uterine nerve ablation

Pharm treatment for dysmen.
Primary:
-FIRSTLINE: NSAIDs
-2-3d before menses onset or cramping, continue for duration of pain
-OCPs
-Vit B6 & Omega-3

Secondary:
-As dx indicates

Consultation/referral recomm. for dysmen.
-Primary: if no improvement after 6mo of tx
-Secondary: if unable to achieve improvement in s/s after 3-6mo

Follow up recomm. for dysmen.
-2mo after initial dx to eval tx

Expected course for dysmen.
Primary: improves w/age/parity
Secondary: based on underlying cause

Possible complications of dysmen.
Primary:
-Anxiety
-Depression

Secondary:
-Infertility from underlying patho

What is the other name for polycystic ovarian dz?
Stein-Leventhal Syndrom

What is PCOD/PCOS?
Complex endocrine condition characterized by:
-hyperandrogenism
-ovulatory dysfunction
-polycystic ovaries

4 phenotypes of PCOD/S:
-Hyperandrogenism/chronic anovulation
-Hyperandrogenism/polycystic ovaries on US but w/ovulatory cycles
-Chronic anovulation/polycystic ovaries w/out hyperandrogenism
-Hyperandrogenism, chronic anovulation, polycystic ovaries

Etiology of PCOD/S:
-Relatively unknown
-Genetic/hereditary link
-Hypersecretion of androgens

Most common endocrinopathy in women of reproductive age:
PCOD/S

Risk factors for PCOD/S:
-Elevated BMI
-Obesity
-DM (I, II, gestational)
-Metabolic syndrome
-Epilepsy/antiepileptic drugs
-Premature pubarche
-Atypical central precocious puberty
-Prenatal complications (high birth wt born to overwt moms, congenital virilization, low birth wt, fetal exposure to androgens)

PCOD/S assessment findings:
-Infertility
-Hyperandrogenism (acne, hirsutism, thinning scalp hair, alopecia, clitoral hypertrophy)
-Ovulatory dysfunction (anovulation, oligo-ovulation, ovarian cysts)
-Wt gain
-High insulin levels
-Acanthosis nigricans
-Depression
-Anxiety

Diff. dx for PCOD/S:
-Cushing’s
-Acromegaly
-Thyroid dz
-Endocrine tumor: androgen-producing ovarian or adrenal tumor
-Prolactin-producing pituitary adenoma
-Adult-onset adrenal hyperplasia
-11 beta-hydroxylase deficiency
-HAIR-AN syndrome (incl. hyperandrogenism, insulin resistance, acanthosis nigricans)
-Endometrial carcinoma
-Drug-induced (steroids, Danazol, Depakote)

Adult diagnostic criteria for PCOD/S
Rotterdam Consensus Criteria:
-Evidence of 2 out of 3: hyperandrogenism, oligo-ovulation/anovulation, polycystic ovaries on US

Adolescent diagnostic criteria for PCOD/S
All 3 criteria according to Rotterdam should be present:
-Oligomenorrhea/amenorrhea present 2y after menarche
-Polycystic ovaries on US should incl. evidence of incr. ovarian size
-Hyperandrogenemia dx’d via lab analysis

Diagnostic studies for PCOD/S
US:
-1 or both ovaries: 12 or more follicles 2-9mm OR incr. ovarian volume >10cm
-ID of endometrial abnorm.

Lab:
-Elev. testosterone
-Sex hormone binding globulin (SHBG) decreased
-Exclusion of other causes of hyperandrogenism: TSH normal, prolactin normal, 17-hydroxyprogesterone (random normal <4, morning fasting level <2) -2hr GTT -Fasting lipids: HDL <50, TG>150

Consider:
-FSH/LH to determine anovulation cause
-Fasting insulin
-Eval for Cushing (24h urinary free-cortisol excretion test, dexamethasone suppression test)

Nonpharm tx for PCOD/S
FIRST-LINE:
-If overwt: lose wt, nutrition counseling

SECOND-LINE:
-Acupuncture
-Hair removal therapy
-Behavioral health

Pharm tx for PCOD/S
FIRST-LINE (preg NOT desired):
-Low dose COC (alternatives: hormonal patch, vag ring)
-Depo

FIRST-LINE (preg desired):
-Metformin
-Clomid
-Femara

SECOND-LINE:
-Aldactone
-Vaniqa
-Proscar

Consult/referral recomm. for PCOD/S
-Infertility specialist
-Endo: insulin resistance or DM
-Behavioral health for anxiety, depression, eating/mood disorders
-Nutritionist: BMI diagnostic of overwt/obesity

Follow up recomm. for PCOD/S
Min. of q6mo or more depending on tx effectiveness

Key visit focus areas:
-Med effectiveness
-SE
-Dz complication
-Wt
-DM screening
-CV risk screening

Possible medical complications of PCOD/S
-Endometrial, breast, ovarian ca
-Hyperlipidemia
-Hyperinsulinemia
-Hyperglycemia
-Insulin resistance
-DM
-CVD
-Infertility
-Obstetrical compl (preeclampsia, gest. HTN/DM)
-Mental health compl. (anxiety, depression)

What is PMS?
Group of >300 symptoms occurring during luteal phase of menses, resolves w/menstruation

What is PMDD?
More severe than PMS
Clinically dx’d using DMS V criteria

Etiology of PMS/PMDD
Unknown
Hypotheses:
-Incr. sensitivity to normal cycling level of estr./progest.
-Incr. aldost./plasma renin activity
Neurotransmitter abnorm. (serotonin)
Twin concordance studies: genetic predisposition?
Pt’s w/PMS have great risk of menopausal s/s

Risk factors for PMS/PMDD
-Younger age a/w more severe symptoms of PMDD
-Higher education
-Heritability
-Incr. life stress, esp. day to day stress
-Past sexual abuse, esp. childhood

Assessment findings of PMS/PMDD
-Swelling
-Breast tender
-Aches
-HA
-Bloating
-Sleep disturb.
-Appetite change
-Poor concentration
-Decr. interest
-Social w/drawal
-Irritability
-Mood swings
-Depression
-Anxiety/tension
-Feeling out of control

Diff. dx for PMS/PMDD
Diagnosis of exclusion:
-Dysmeno.
-Autoimmune disorders
-DM
-Anemia
-Hypothyroid
-Endometriosis
-Depression
-Dysthymic disorder

Diagnostic criteria for PMS
ACOG requires 1 of at least 10 specified symptoms (6 affective, 4 somatic symptoms):
-Occurs 4d prior to menses in 3 prev. menst. cycles
-Remission during menstr. flow
-Doesn’t occur before cycle day 13
-Symptoms cause significant impairment or distress, cannot be accounted for by another dx or med
-Reported symptoms should be prospectively confirmed by daily sympt. rating for 2 or more menstr. cycles

Affective symptoms of PMS
Depression
Angry outburst
Anxiety
Confusion
Social withdrawal

Somatic symptoms of PMS
Breast tenderness
Bloating
HA
Swelling

Diagnostic criteria for PMDD
DSM V:
-5 or more sympt. (at severe level premenst.) in final week before start of menses w/improvement few days after onset of menses
-One must be core sympt.

Core symptoms of PMDD
-Marked affective lability
-Marked irritability/anger or incr. interpersonal conflicts
-Marked depressed mood, feelings of hopelessness, self-depreciating thoughts
-Marked anxiety, tension, feelings of being on edge

Other symptoms of PMDD
-Decr. interest in usual activities
-Subj. difficulty in concentration
-Lethargy, easily fatigued, marked lack of energy
-Marked appetite change (overeating, specific food cravings)
-Hypersomnia, insomnia
-Sense of being overwhelmed, out of control
-Physical symptoms (breast tend., swelling, joint/musc. pain, bloating, wt gain)

Diagnostic studies for PMS/PMDD
-Premenstr. Symptom Screening Tools (PSST) for adults
-Premenst. Symptom Screening Tool revised for adolescents (PSST-A)
-Labs: CBC, TSH

Nonpharm tx of PMS/PMDD
-Menst. diary x2 cycles to establ. pattern
-Dietary mods (decr. caff., Na, ETOH, choc., high fat foods; incr. complex carbs)
-Lifestyle mods (reg., aerobic exercise x30min 3-4 times/wk, smoking cessation, adequate sleep/rest, stress reduction)
-Acupuncture
-Support groups
-Cognitive behav. therapy
-Bilat salpino-oophorectomy (only if severe, persists after all other tx options failed, other dz’s ruled out)

Pharm tx for PMS/PMDD
-COCs (Yaz)
-SSRI (Sarafem, Zoloft, Paxil)
-NSAIDs (Motrin)
-Diuretics (Aldactone)
-Vit/mineral suppl: B1/6, Ca, Mg, Vit E
-Herbs: vitex angus-castus, gingko biloba, Crocus sativus, ginger
-CAUTION: anxiolytics (Buspar)/GnRH agonists

Consult/referral recomm. for PMS/PMDD
-If refractory to tx
-If pt prefers tx w/integrated medicine or cogn. behavioral therapy

Follow up recomm. for PMS/PMDD
-Re-eval in 2mo, review menst. diary, adjust tx plan if lifestyle/dietary/OTC meds ineffective
-Follow up in 2mo or 2 cycles after tx change, then q3-6mo

Expected course of PMS/PMDD
-Usually adequately controlled
-PMS symptoms can continue after hysterect.

Possible complications of PMS/PMDD
-Severe depression
-SI
-Social isolation
-Economic burden from decr. work productivity/incr. absenteeism

Vulvar ca description
-Ca of outer aspect of female genitalia (incl. labia majora/minora, introitus, clit)
-Squamous cell carcinoma most common (90%)
-May/may not be caused by HPV

Etiology of vulvar ca
-Advanced age
-Smoking
-Vulvar intraepithelial neoplasia (VIN)
-HIV
-Other urogenital ca
-Melanoma

Assessment findings in vulvar ca
-Preinvasive lesions: flat, hyperkeratotic, may vary in color (white, pink, brown)
-SCC: maybe Lg lesion that grows outward from epithel. surface or as small ulcer

Differentials for vulvar ca
-Epidermal inclusion cysts
-Condyloma
-Vulvar dermatoses
-Seborrheic keratosis

Diagnostics for vulvar ca
Any suspicious lesion should be bx’d

Prevention of vulvar ca
-HPV vax
-Smoking cessation
-Early detection of other ca

Nonpharm tx of vulvar ca
-Surgical intervention dependent on clinical staging
-Chemoradiation may be used in conjunction with surgery for Stage III/IV

Consult/referral recomm. for vulvar ca
Gyn or onc if ca suspected

Follow up recomm. for vulvar ca
-No evidence to support any one follow up sched.
-Common follow up sched: q3mo x1y, q6mo x1y, then yearly

Expected course of vulvar ca
-Hasn’t spread beyond vulvar region: 86% 5y survival rate after dx/tx

Possible complic. of vulvar ca
-Metastisis
-Death

Vulvodynia description
Chronic vulvar discomfort
Usually described as burning
Lasts >3mo in absence of infection, inflam., neoplasia, neuro disorder

Classifications of vulvodynia
Location:
-Localized: only a portion of vulva but most often involving vestibule
-Generalized: diffuse involv. of entire vulva

Provocation:
-Provoked: any physical touch/stimulation that elicits pain
-Unprovoked: spontaneous pain w/out touch/stim.
-Mixed: varies w/or w/out touch/stim.

Etiology of vulvodynia
Unknown (prob. multifactorial, unlikely single cause)
Proposed causes:
-Embryonic derivation
-Chronic/recurr. inflam./infection
-Genetic immune factors
-Neuropathic changes
-Hormonal changes
-H/o HPV
-Oxalates

Incidence of vulvodynia
From adolescence to menopause

Risk factors for vulvodynia
-Unprotected sex
-Scented detergents/hygiene prods.
-Non-breathable clothing/fabric
-Improper hygiene
-Other infect. prevention methods

Assessment findings for vulvodynia
Mons pubis, labia majora/minora, vestibule, perineum, clit:
-Burning
-Stinging
-Itching
-Aching
-Soreness
-Throbbing
-Irritation
-Rawness
Dyspareunia
Usually no visible abnorm., but maybe erythema/lichen sclerosis at pain/itching site

Differentials for vulvodynia
-Urogenital infect.
-Skin dz
-Vulvar/vag. neoplasia
-Comorbid chronic pain disorders
-Sexual dysfunct.

Diagnostics for vulvodynia
-Dx based on clinical presentation
-Bx/vulvar colposcopy to rule out dermatoses/other pre-malig./malig. conditions
-Cotton swab test to ID painful areas
-Vag exam w/wet prep/yeast cx to rule out vag/vestib. infecti.
-UA w/C&S to rule out UTI

Prevention of vulvodynia
-Avoid underlying cause if possible

Nonpharm tx of vulvodynia
Vulvar care:
-Cotton underwear during daytime, no underwear at night
-Avoid irritants (scented deterg./soap/hygiene prods).
-Bathe in mild baking soda soln w/lukewarm water
-Apply veg/olive oil to vulva after bath to retain moisture
-Ice/cool gel packs to vulva 2-3min at a time

Use lube during sex
Psych support/counseling/therapy
Acupunct./biofeedback/PT/hypnotherapy/Ntg/botox
Low oxalate diet w/Ca citrate suppl. (may help, not well-documented)
Surg. may be indicated in select cases (only after other tx unsuccessful)

Pharm tx for vulvodynia
-Lidocaine 5% ointment
-A&D w/zinc oxide
-Amitriptyline 2% w/baclofen 2%
-PO amitriptyline
-Desipramine
-Gabapentin
-Pregabalin
-Topiramate
-Hydroxyzine
-0.5% bupivacaine w/hydrocort

Consult/referral recomm. for vulvodynia
For surg. eval (vestibulectomy) after prev. tx failed

Follow up recomm. for vulvodynia
Monthly to eval response to tx w/realistic expect. for relief

Expected course for vulvodynia
-Complex disorder
-Rapid resolution unusual
-Decreased pain may take wks-mos
-No single tx successful in all women

Possible compl. with vulvodynia
-Physical/psych distress
-Vaginismus
-Sexual arousal disorder

Amenorrhea description (primary)
Failure to reach menarche by 15yo (w/norm. secondary sexual charact.) or 13yo (no secondary sexual charact.)

Amenorrhea description (secondary)
Cessation of reg. menses x3mo or cessation of irreg. menses x6mo

Etiology of amenorr.
Primary:
-Maybe result of chromos. irreg. leading to primary ovarian insufficiency/anatomic abnorm. (Turner, Mullerian agenesis)

Secondary (most cases attributed to):
-PCOS
-Hypothalamic amenorr.
-Hyperprotlactinemia
-Ovarian failure

In all amenorrhea pts, exclude:
Preg

Risk factors for amenorr.
-Anovulation
-Hormone replacement
-Obesity
-Nulliparity
->35yo
-DM
-Personal/family h/o coag disorder
-Liver disorder
-Anticoag/chemotherapy

The presence of breast devel. in amenorr. means there has been:
previous estrogen actions

Excessive testosterone secretion in amenorr. is suggested usually by:
Hirsutism (rarely by incr. musc. mass, other signs of virilization)

Phys. exam findings in amenorr.
-Examine thyroid
-Anthropomorphic measurements: ht, wt, waist circum., %body fat, etc.
-Dysmorphic features (webbed neck, short stature)
-Alopecia
-Hirsutism
-Acne
-Striae
-Buffalo hump
-Central obesity
-HTN

Pelvic exam findings in amenorr.
-Absense/abnorm. cervix/uterus
-Clitoromegaly
-Vag septum or imperforate hymen
-Red/thin vag mucosa

Differentials for amenorr.
Anatomic abnorm.:
-Mullerian agenesis (blind/absent vag. w/breast develop.)
-Imperforate hymen
-Transverse vag. septum (blind/absent vag. w/breast develop.)

Primary ovarian insuff.:
-From idiopathic, empty sella syndrome, or autoimm./infiltrative process
-Turner syndrome (charact. by chromos. pattern of 45, X)

Hypothal./Pituitary:
-Functional hypothal. amenorr. stemming from stress, wt loss, excessive exercise, disordered eating

Serum prolactin elevation:
-Pituitary adenoma
-Hypothyroid
-Mass/lesion

CNS etiologies:
-Previous CNS infection
-Trauma
-Autoimm. destruction of pit. gland

Others:
-PCOS
-Preg/contracept.
-Thyroid/adrenal dz

Diagnostics for amenorr.
Labs:
-Beta hCG
-CBC
-FSH
-CMP
-LH
-Prolact
-TSH
-Progest. challenge test
-Testost.
-Estradiol
-Genetics (karyotype, phenotype)

Imaging:
-If genital exam not feasible, abd US may be useful to confirm presence/absence of uterus
-US or MRI: key to dx if anatomic abnorm.
-Hysteroscopy

Nonpharm tx of amenorr.
-Surgical correct. of struct. abnorm.
-Surg. resection of tumors as necess.
-Lifestyle mod. if wt, stress, diet related

Pharm tx of amenorr.
Depends on etiology:
-OCPs: re-estab. menst. cycle
-Meds to initiate ovulation
-Symptomatic management of PCOS
-Estrog. replacement: re-establ. menstr. cycle in genetic abnorm. or primary ovarian insuff.

Consult/referral recomm. for amenorr.
Endocrine:
-Findings consistent w/thyroid, hyperprolact., PCOS, Turner

Surg/neuro:
-Findings consistent w/adenoma, pit. tumor

Follow up recomm. for amenorr.
Depends on etiology

Expected course of amenorr.
Depends on etiology

Possible complic. of amenorr.
-Incr. risk of ca from anatom. abnorm.
-Bone dens. loss if decr. estrog. levels
-Osteoporosis
-Infertil.
-Small stature

What is condyloma lata?
Syphilitic wart

PID description
Inflamm. disorder of gyn tract cuased by ascent of microorganisms from vagina and endocervix to uterus, fall. tubes, ovaries, contiguous struct.

Etiology of PID
-N. gonorrhea
-C. trachamatis
-Anaerobic organisms
-Mycoplasma genitalium
-Gardnerella vaginalis
-H. influenza
-Gram-neg rods
-Strep agalactiae
-Regardless of organism: view and tx as mixed polymicrobial infection (but chlamyd/gonorr. are common)

Highest rates of PID occur among:
African-American & Black Caribbean (15-25yo)

Risk factors/causes of PID
-IUD
-Sex
-<25yo
-Adolescence
-Mult. sex partners
-Previous STD/PID or partner w/STD
-Douching
-Intercourse during menses

Assessment findings in PID
-Asymptomatic
-Sympt. usually begin during/within 1wk of menses
-Unusual/new onset abnorm. uterine bleeding, dysmenorr. w/or w/out menorrhagia
-Low abd tenderness or pain/rebound tenderness
-Fever
-Malaise
-Vag discharge/lesion
-Urinary discomf.
-N/v
-Cervical motion tenderness/uterine tenderness
-Dyspareunia
-Subclinical PID
-Adnexal tenderness

Differentials for PID
-Appy
-Ectopic
-Ovarian cyst/ruptured
-Endometriosis
-Ovarian torsion/tumor
-Constipation
-IBD

Diagnostics for PID
-Preg. test (START HERE)
-CBC (WBC >10,500; result alone is unlikely to alter management plan)
-Wet mount
-Pelvic US
-Pelvic CT/MRI
-Complete screen for STDs
-Hepatitis serology
-UA/cx

Diagnostic criteria for PID
Suggested criteria (sufficient for empiric therapy):
-Pelv/low abd tend/pain AND
-Cerv. motion tend. OR
-Adnexal tend. OR
-Uterine tend.

Additional support:
-Temp >101F
-Abundant numbers of WBC on saline micro of vag fluid
-Purulent material obtained w/culdocentesis
-Abnorm. cerv/vag discharge, cerv friability
-Elev. CRP/ESR
-Adnexal mass

Definitive:
-Histopathologic endometritis on bx
-TVUS or other imaging show thick fluid-filled tubes w/or w/out free pelv. fluid or tubo-ovarian complex.
-Doppler shows cause (like tubal hyperemia)
-Laparascopic findings consist. w/PID

Prevention of PID
-Condoms/spermicide provide some degree of protect.
-OCPs have been shown to decr. incidence of PID

Nonpharm tx of PID
-Insuff. evid. to support removal of IUD in pt’s dx’d w/acute PID. Close monitoring is mandatory.
-Abstinence until tx completed
-Eval/tx sex partners
-Screen reg. for STDs in at risk pt’s

Pharm tx of PID
No specific regimen considered superior because tx is empiric until cx confirms organism. Abx coverage for chlamyd./gonorr/anaerobes/gram-neg rods/strep should be included.

Outpt:
-Cefoxitin (Mefoxin) 250mg IM x1 dose PLUS
-Doxy 100mg BID x14d WITH OR WITHOUT
-Flagyl 500mg BID x14d

Optional regimen A:
-cefoxitin 2g IM x1 AND probenecid 1g PO x1 PLUS
-doxy 100mg BID x14d WITH OR WITHOUT
-flagyl 500mg BID x14d

Optional regimen B:
-Other parenteral 3rd gen. ceph. PLUS
-Doxy 100mg BID x14d WITH OR WITHOUT
-Flagyl 500mg BID x14d

Preg./lact. considerations in PID
Hospitalization required in preg.

Consult/referral recomm. for PID
Hospitalize for:
-Unable to exclude surgical emerg.
-Preg.
-No clinical response (at 72h)
-Unable to follow up
-Tubo-ovarian abscess
-Severe n/v or high fever

Follow up recomm. for PID
-Close obs of clinical course w/re-eval in 72h (sooner if sympt. worsen)
-Test of cure in 3-6mo after tx
-Screen for STD incl. HIV

Expected course of PID
-Good progn. if tx early w/effective tx
-Poor progn. related to poor compliance & repeated infect.
-Pt’s w/pre/coexist. conditions need closer obs/maybe hosp. (STD, preg., IUD, prev. PID)

Poss. complic. from PID
-Recurr. infection
-Incr. risk of ectopic
-Infertility
-Sepsis
-Chronic pelvic pain
-Tubo-ovarian abscess
-Perihepatitis (Fitz-Hugh Curtis syndrome)

Atrophic vaginitis description
Part of a group of symptoms w/decr. estrog./other sex steroids involv. labia majora/minora, clit, vestibule/introitus, vag, urethra, bladder.

Etiology of atrophic vaginitis
Estrog. deficiency causing physiological, biological, clinical changes in urogenital tiss. secondary to:
-Menopause
-Oophorectomy
-Pelvic radiation

Assessment findings of atroph. vaginitis
-Vag dryness/burning/itching/irritation
-Atrophy/absence/decr. vag rugae
-Pruritis
-Blood-tinged vag discharge
-Bleeding after intercourse
-Erythematous/petechial patches on vag mucosa
-Dyspareunia
-Urinary incont./urgency
-Dysuria
-Recurr. UTI

Differentials for atroph. vaginitis
-Candida vaginitis
-Bact. vaginosis
-Trich or other STD
-UTI

Diagnostics for atroph. vaginitis
-Wet prep: normal vag flora
-FSH to confirm menopause (will be incr.)
-Estradiol level to measure circulating estrog. level (decr. in menopause)
-Pap/mammo before initiating estrog. therapy
-UA/cx

Prevention of atroph. vaginitis
-Hormone replacement therapy in estrog. deficient states: personal decision which should be made after consult. w/healthcare prov.

Average age of menopause
52yo

Nonpharm tx of atroph. vaginitis
-Water-soluble lubes/moisturizers before intercourse
-Cool baths/compresses
-Cotton underwear
-Kegels/PT for incont.

Pharm tx of atroph. vaginitis
-Maybe low dose PO estrog. daily
-Progest. should be added to PO estrog. for women who haven’t had hysterect.
-Low dose vag estrogen therapy
-PO estrog. agonist/antagonist
-Abx for UTI

Absolute contraind. for estrog. replacement therapy
-h/o breast ca
-Undx vag bleeding
-Carcinoma
-Active liver dz

Options for incont. in atroph. vaginitis
-Surgery: midurethral sling
-Botox
-Sacral neuromodulation tx

Consult/referral recomm. for atroph. vaginitis
Not usually required

Follow up recomm. for atroph. vaginitis
-Re-eval 1-2mo after starting drug tx
-Annual phys. exam

Expected course of atroph. vaginitis
-Sympt. should resolve in 1-2mo after tx started
-Most will be relieved by estrog. replacement ther.

Possible complic. of atroph. vaginitis
-Secondary infect.
-Vag fissures/ulcerations

A mother presents with her 3 month old child and complains of goopy eye that is sometimes matted shut after naps. Upon exam the conjunctiva is clear with some mild crusting to eye lashes. What is the diagnosis and patient teaching?
Probable lacrimal duct obstruction.

-The baby will likely grow out if it somewhere between six and 12 months.
-You can do a daily lacrimal duct massage and warm compresses to help open up the duct.
-Watch for Periorbital cellulitis or bacterial conjunctivitis.

Nine-year-old patient presents with tender, swollen, erythemic bump to eyelid. What is the diagnosis and patient education?
Hordeolum (stye)

-warm moist compress

  • scrub eye lashes with baby shampoo BID
  • eye will feel better when stye pops, if doesn’t will refer to optho.

What is blepharitis?
inflammation of the eyelid

  • caused by blocked oil gland, allergies, or excessive bacteria

What is the treatment for blepharitis?
Cleanse with baby shampoo TID

  • Warm, moist compress
  • resistant can prescribe Doxy or Tertracycline but not to pediatrics who don’t have adult teeth, can cause discoloration. Pediatrics can have Bactrim.

A 9 year old patient presents with acute swelling and erythema to eyelid with flaky, scaly debris to eyelid margins. Patient reports gritty, burning feeling to eyes. She has already attempted warm compresses and washing eyelids with baby soap with minimal relief. What is the diagnosis and treatment?
Patient has anterior blepharitis

Treat with erythromycin ointment, ointment has longer contact with eye.

Can use azithromycin ointment x 4 weeks for long term treatment

Can use doxycycline or tetracycline for chronic in kids >8 years or Azithromycin oral for kids <8

What organism is likely to cause viral conjunctivitis?
-Adenovirus
-HSV
-Herpes

What organism is likely to cause bacterial conjunctivitis?
-Staph aureus
-strep pneumoniae
-H. Influenza

A 10-year-old patient presents with unilateral conjunctivitis with watery discharge x 2 days along with itching of the eye. Mother denies any other associated symptoms. What is the diagnosis and treatment?
Viral Conjunctivitis

Patient should apply cold compresses for comfort

*Bacterial conjunctivitis has purulent discharge.

What is herpes keratitis?
Herpes viral infection to the eye caused by HSV.

A 19-year-old male presents with blurred vision, watery discharge, photophobia, and eye redness. Upon exam you note fern-like lines under the woods lamp. What is the diagnosis and tx?
Herpes Keratitis

Tx: Antivirals

A 16-year-old female presents with complaints of sudden onset of red, watery, and painful left eye. She reports possible foreign body to eye. Upon inspection you visualize a n erythemic lesion to the eye. When using woods lamp you note a small superficial laceration. What is the dx and tx?
Corneal abrasion, refer to optho emergently or send to ER.

A mother brings her nine-year-old overweight son to the clinic with complaints of pain to the groin, knee, and right hip. She reports patient has been walking with a limp for the last couple days but unable to ambulate today. Mother denies any recent trauma. Upon inspection there is external rotation of the right foot. What is the dx and tx?
Slipped capital femoral epiphysis

Refer to ER, will need surgery.

-most common in adolescent, obese males.

Father presents with his four-year-old son and states that patient has been limping since this morning and was crying all night complaining of left hip pain. Upon exam you appreciate decreased range of motion of left hip but no external rotation or signs of trauma. What is the probable diagnostics, and treatment?
Toxic transient syndrome- most common cause of limp in peds.

Order: CBC, ESR, and X-ray. WBC and ESR will be elevated. X-RAY will be normal.

-advise father that will resolve in 2-6 weeks.

  • patient needs bed rest, NWB to left leg, and NSAIDS.
  • return if develops fever, swelling, pain that is unrelieved by rest or medication

What degree is scoliosis is considered functional?
5-10 degrees, continue to monitor.

What degree of scoliosis is considered mild?
<20 degrees, monitor every 3-4 months

A 13-year-old female presents to the office after having an x-ray order by you for her scoliosis. The reading on the x-ray reports a 30° curvature. What is the treatment plan?
Milwaukee brace for 23 hours per day

Back exercises

At what point does scoliosis become a surgical case?
Curves greater than 45°

A 16-year-old female presents to the office with complaint of right knee pain with intermittent swelling. Upon questioning patient endorses pain is worse when walking up or down stairs, kneeling, squatting, or sitting cross legged. Patient denies any trauma but reports being active in cheer. What is the dx and tx?
Chondromalacia petella “runners knee”

RICE and PT consult to strengthen quadriceps.

A five-year-old boy presents with his father for a complaint of pain in the right knee that radiates to right groin. Father endorses that patient has been ambulating with a intermittent limp, especially after exertion, and is refusing to play baseball due to pain. Father denies any trauma. Upon inspection you notice that patient is small for his age. There is no signs of infection or trauma and no external rotation but pain when rolling the leg internally. What is the dx and tx?
Legg-Calve Perthes disease- hip joint slips out of socket. Affects boys 4-10

Dx: X-ray (AP pelvis and frog-leg lateral views)

Tx: refer to ortho, NWB, Rest, NSAIDs, PT
LAST RESORT: surgery <6 years old or casting

A newborn presents to the office for a well-check, you are concerned about Cystic Fibrosis, what test should be ordered?
1st- IRT
2nd- if IRT positive then sweat test or gene analysis.

What is systems of cystic fibrosis?
-persistent diarrhea
-poor weight gain
-chronic cough
-respiratory problems

What are findings using jones criteria for rheumatic fever?
Strep infection, carditis, and erythema marginatum

Concurrent otitis media & conjunctivitis is likely due to which organism?
a. streptococcus pneumoniae
b. haemophilus influenza
c. moraxella catarrhalis
d. staphylococcus aureus
Haemophilus influenza

What is the appropriate management for a child who presents with acute wheezing and/or cough?
Admin. SABA

A 10-month-old child has been diagnosed with gastroenteritis. He attends daycare, what is the most likely cause of illness?
Rotavirus

If a child presents with a “pop” in the knee, what is that indicative of?
Anterior cruciate ligament tear

What is the management of a newborn diagnosed with developmental dysplasia of the hip?
Pavlik harness

An essential test in evaluation of Kawasaki disease?
Echocardiogram

A 4-week-old presents to your office in January with a 1 week history of nasal congestion and occasional cough. On the evening prior to visit patient developed a fever of 102, refused to breastfeed, and had proximal coughing and noisy, labored breathing. On exam you note an Ill appearing infant who is lethargic with tachypnea. What is diagnosis and treatment?
Bronchiolitis- commonly caused by RSV.

treat symptoms unless resp distress then send to ER.

What education should be included for parents of a febrile seizure child?
Child May experience repeated seizures.

What is the Barlow maneuver?
posterior dislocation w ADduction

(think: barlow – back, ortolani – open)

What is the Ortolani maneuver?
anterior reduction w/ ABduction

A mother of a two week old presents with complaints of “rash” to baby’s face. Upon inspection you notice faint pink patches with feathery borders on the forehead between the eyebrows. What education would you provide for the mother?
These are Nevus Simplex also known as stork bites. They will Fade in the first few years of life. No treatment is necessary.

A mother of a newborn presents with concern over discoloration on her child’s face. Upon inspection you notice a large erythemic macular patch to the baby’s cheek and nasal fold. What is the dx and education?
port wine stain

-this is a permanent birthmark and could turn darker as a child grows.
-when the child reaches adulthood you can pursue laser treatment to help fade the birthmark

A mother presents for a newborn check up and upon inspection you noticed several café au lait spots (macular hyperpigmentated). You begin to count and find more than six spots, what should you concerned for?
Neurofibromatosis- refer to neurologist.

A mother presents with a one month old for concerns of a large, raised, reddened growth to her belly button. She states it has been present since birth but is worsening. What is dx. And tx?
Hemangioma- Common vascular birthmark made of extra blood vessels in the skin.

No treatment needed

A mother of an African-American child presents to the office for a newborn well visit. She reports her son was born with Mongolian spots. When would Mongolian spots be concerning?
If the spots grow or worsen would be a concern for lysosomal storage disease.

A mother of a two-year-old presents and states the patient has had excessive anal itching at night time and has been irritable due to lack of sleep. What is the presumed diagnosis and treatment?
Pin worms

Pinex

Whole family needs treatment, encourage good hand hygiene, no nail biting, wash bedding.

A child is brought by his mother to your office with a pink rash that first appeared on the face and then made its way down the trunk and extremities. what is the most likely cause?
Rubeola

What is the most common causative organism a fifth disease?
Parvovirus 19

When can a child go back to school as they are no longer contagious post chickenpox?
Once the lesions have crusted

When can a child go back to school post roseola infection?
Once the fever has subsided

A child presents with lesions on his hands, feet, and mouth. What should you be most concerned about?
Dehydration

The patient has hand, foot, and mouth and will decrease oral intake due to painful lesions to mouth.

Fifths disease is also known as?
erythema infectiosum

MMR and VZV schedules?
12 months and 4 years

Name the viral exanthem that has a rash which is small white spots with a reddened background that occur on the inside of the cheeks?
Koplix spots seen in measles

Which two viral exanthems have cephlocaudal spread?
Rubella and Measles ( head to body spread)

Patient presents with a generalized rash, the mother tells you the new lesions appear daily. Upon inspection the rash is vesicular, pruritic, and covers all the body. Which organism is most likely to cause?
Varicella Zoster (chickenpox)

Patient presents with a slapped cheek appearance and a lacy reticular rash on the body. Reports fever previously then developed rash. What is the dx and tx?
5ths disease (parvovirus-19)

Treat symptoms, rash will disappear in 2-3 weeks.

AVOID IN PREGNANCY!

When are lead levels checked?
12 and 24 months

Positive >5

What lead level needs treatment?

45

What are red flags for headaches in pediatrics?
Sudden onset
Worsening patterns
Systemic illness
Focal neurological signs
Papiledema

Treatment for migraines in pediatrics
Beta blockers, antidepressants, anticonvulsant, CCB

Mother presents with 1 year old who appears to have Genu Varum. What teaching would you provide?
Genu Varnum (bow leg) is normal up to 2 years of age. Your child’s legs are likely like that because they had to rotate to fit into your uterus.

  • if they remain by 3, we will consider other disease processes or refer to ortho.

Causes of bow legs?

  • Blounts disease- growth disorder

Mother presents to the clinic with her son who has cystic fibrosis. What education should the nurse practitioner provide for the mother as far as expectations?
-Child will be above 50th percentile for weight and height
-diet should include high fat and whole milk
-there should be no person-to-person interactions with other cystic fibrosis patients

Mother presents to the clinic with her nine-year-old child who was hit in the head with a baseball bat. Upon exam the patient is AAOX3 But is complaining of a headache, fatigue, and slight nausea. Mother denies any loss of consciousness. What is the diagnosis and treatment?
Traumatic brain injury/concussion

TX: watchful waiting, have child return in 1-2 weeks for recheck.

  • gradually return to non-sports activities after no more than 2 to 3 days of rest
    -May use Tylenol for headache

What are red flags following a traumatic brain injury?
-watery discharge from nose or ears
-vomiting more than two times
-seizures
-memory loss
-unequal pupils
-increased drowsiness

What should pediatric patients be started on if they are positive for sickle cell?
-referral to pediatric hematology
-begin penicillin VK 125 mg by mouth starting at 3 months. (Preventative for peds under 5 or high risk)
-Do CBC, Retic count, and G6PD

  • vaccinations

What additional preventative action should patients with sickle cell have?
-routine blood pressure monitoring
-Transcranial Doppler’s up to 16 years old
-Retinal exam starting at age 10

What vaccination should sickle cell patients have and why?
-PCV13 (2m, 4m, 6m, 12m)
-PPSV23 (24m, 5y-7y)
-influenza (6m starting..)

They are at higher risk for infection that could be deadly.

A four month old infant’s Mother calls and states the patient has a fever of 101.8 rectally. The mother reports that the patient has received her penicillin prophylaxis that was prescribed for sickle cell. There is no respiratory distress and the patient has good oral intake and normal urine output. What advice would you give the mother at this time?
The mother should be advised to take the patient to the emergency department immediately for initiation of IV antibiotics and further evaluation

Out of the following children who should be seen immediately?

A.) A four month old who has had a fever of 101.2 x 2 days but is still playing and has good oral intake.

B.) A two month old with a fever of 100.6

C.) A 13 year old with a fever of 103

D.) A 3 year old with a fever of 101.1
B

Babies <3 months with a fever greater than 100.4 should be seen immediately

Exclusively breast-fed infants should be started on what at four months of age?
Iron supplementation

Iron-fortified cereal and/or purred meats at 6 months

What education should be provided for families of children taking iron supplements?
-avoid administering with meals or milk
-vitamin C juice enhances absorption
-Child stores will turn black
-Any dental staining can be removed with dental cleaning

A mother presents with a six-year-old male with complaints of fever times one week and rash. Upon exam you find conjunctival injection, red, tender, edematous hands and feet, cracked red dry lips, cervical lymphadenitis, and a red strawberry tongue. What is the diagnosis and treatment?
Labs: CBC, CMP, ESR, and CRP

Kawasaki Disease- vasculitis

Admit to hospital, needs: IVIG, ASA (antiplatlet therapy), antiplatlet

*no live vaccinations x 11 months post IVIG.

rheumatic fever
A bacterial infection that can be carried in the blood to the joints from group A strep

Jones Criteria to help identify

What is the treatment for rheumatic fever?
First line: PCN

If allergic to PCN: azithro

Prednisone
ASA
No prophylaxis for endocarditis in rheumatic fever

What is the difference between croup and Bronchiolitis?
Croup targets the upper respiratory track, bronchitis threatens the small and medium lower respiratory tract passages

Croup: harsh barky (seal-like) cough, inspiratory stridor, hoarseness.

Bronchiolitis: clear rhinorrhea, cough, tachypnea, wheezing, retractions, resp distress. Poor appetite and feeding. Think RSV!

A patient presents with asthma, when questioning their SABA use patient reports using it > 2 days per week but not daily. What degree of asthma is this and what is the treatment?
Mild

Use SABA and prescribe low dose ICS
Ensure proper SABA use

A patient presents with asthma, when questioning their SABA use patient reports using it daily, mostly in the AM, but not multiple times per day. What degree of asthma is this and what is the treatment?
Moderate

Continue SABA, prescribe medium dose ICS and refer to pulmonologist.

A patient presents with asthma, when questioning their SABA use patient reports using it multiple times a day and has extreme limitations to activity. What degree of asthma is this and what is the treatment?
Severe

Continue SABA, prescribe high dose ICS and LABA or Montelukast. Refer patient to pulmonologist.

A patient presents with asthma, when questioning their SABA use patient reports using it maybe 1 time per week, has no limitations to activity. What degree of asthma is this and what is the treatment?
Intermittent

continue SABA use

A three-year-old female presents with her mother for a complaint of diarrhea times seven days. Mother denies any nausea, vomiting, fever, or abdominal pain. Upon assessment skin is warm and dry with no turgor. What is the diagnosis and treatment?
Gastroenteritis

-maintain hydration
-maintain nutrition

-antidiarrheals are not recommended
-Pepto-Bismol not recommended due to concerns for reye syndrome

A mother presents with her one-month-old infant and states she has concerns for frequent spitting up. Mother reports spitting up approximately 1oz of formula shortly after eating, no distress during episodes. When asked, mother denies any cyanosis or choking. Reports infant is eating 4oz every 2 hours. What is diagnosis and treatment?
Possibly GERD

Nurse practitioner should educate mother to decrease feedings, one month old infant should be eating about 4 ounces every four hours.

-mother should be educated to burp more often during feedings and keep infant upright for longer after eating.

-GERD can be caused by immaturity of the lower esophageal sphincter, patient should grow out of it.

*check growth chart, should be normal progression. KEY!

If this all fails, could start patient on Pepcid. PPI is second line.

What is cholesteatoma?
a tumor-like mass of scaly epithelial tissue and cholesterol in the middle ear. Associated with AOM.

A 15-year-old male presents to the clinic with complaints of feeling fullness in one ear, dizziness, fluid discharge with foul smell, and decreased hearing. Upon assessment you notice a tumor like mass to the tympanic membrane. What is the diagnosis and treatment?
Cholesteatoma

Tx: Refer to ENT for surgery

A 15-year-old male presents with his mother with complaints of sudden onset of unilateral scrotal pain with nausea and vomiting. Upon assessment you note redness, swelling, and warmth to the involved scrotum. What is the diagnosis and treatment?
Testicular Torsion

*Blue dot sign- subtle blue mass that is visible through scrotal skin.

Tx: send to ER for surgery- EMERGENCY!

A three-year-old male presents with his mother for a complaint of abdominal swelling, pain, nausea and vomiting, loss of appetite, and fever. You get a UA which shows blood in urine. Upon assessment you palpate an abdominal mass. What is the possible diagnosis and treatment?
Wilm’s tumor (common in peds <5 years and African American)

Dx: chest and abd CT
ABD ultrasound

Management: Oncology

*common findings in wilms: Aniridia (iris forms partially)
Hemihypertrophy: one side of body is larger than other.

What illness is associated with Aniridia and Hemihypertrophy?
Wilm’s tumor

What are classic symptoms of Turner syndrome?
-short stature
-short neck with webbing- CLASIC

  • hip dysplasia
    -scoliosis
    -delayed puberty

What should a nurse practitioner monitor in a patient with confirmed Turner Syndrome?
-monitor growth
-nonverbal
-annual hearing exam, reoccurring AOM
-ongoing vision. Strabismus
-monitor BP
-annual thyroid screening
-celiac disease screening

A mother of a patient with confirmed Turner Syndrome is concerned that her daughter has not started her menstrual cycle at 17, what education would you provide?
This is normal, kids with this disease process have delayed puberty.

What are primary care issues for patients with Down syndrome
-hypothyroid screening
-congenital heart disease
—cataracts
-duodenal atresia
-neuro conditions
-Celiac disease
-Leukemia

What education would you provide to a family of a 3 year old diagnosed with UTI?
-increase fluid intake
-schedule voiding times
-avoid tight pants
-avoid bubble bath, sitting in soapy water, and perfumed soaps
Avoid caffeine, carbonated beverages, chocolate, citrus, ETOH, and spicy foods.

Mother brings in 9 month old for well check. On exam you notice that patient has exaggerated reflexes, rigid limbs, and involuntary movements. What is the diagnosis and treatment?
Cerebral Palsy

Dx: MRI

Management: PT/OT, anti-constipation medications.

What is hypospadias?
abnormal congenital opening of the male urethra on the undersurface of the penis

What is cryptorchidism?
undescended testes

A 6 month old presents for a well-visit, upon assessment you note continued cryptorchidism. What is treatment?
Refer to surgeon by 9-15 months

A 9 month old presents for a well-visit. On exam you note scrotal swelling that doesn’t appear to bother the patient. On further investigation, the scrotum is equal bilaterally and skin is pink and warm. What is diagnosis and treatment?
Hydrocele

Tx: should resolve on its own, if persistent for more than 1 year will refer to surgery.

A 1 month old 2 weeks post circ. presents with complaint of red irritated head of penis. Mother reports seeing purulent discharge when changing diaper with fowl smell. What is the diagnosis and treatment?
Balanitis- common in uncircumcised and post circ.

Bactrim orally and bacitracin ointment.

A 7 year old female presents with complaint of fever, weight loss, and joint pain. Upon reviewing her chart you note she has been seen multiple times this year for reoccurring infections. What is the differential diagnosis and possible treatment?
Leukemia- common childhood cancer

Dx: CBC, peripheral smear, and bone marrow biopsy

A 13 year old female presents with complaint of nasal congestion, yellow nasal discharge, fever, and hyposmia (decreased smell) x 10 days. Upon exam patient has maxillary and facial tenderness. What is diagnosis and treatment?
Bacterial rhino-sinusitis

Amoxicillin- low risk patient
Augmentin- daycare, recent antibiotics use, mod/sev illness.
Rocephin 50mg/kg IM.
3rd gen cephalosporin CEFdinir, CEFposoxime, CEFuroxime

A mother presents with newborn for first check-up, when reviewing the chart from birth you discover that the baby was found to have a atrial septal defect. What symptoms do you expect to see? What education can be provided to the mother?
Patient will most likely be asymptomatic, hole between atriums didn’t close like it was supposed too.

Will hear split S2 heart sounds

Education: small ASD will close on its own, watchful waiting. Medium to large ASD will require surgery.

What is pulmonary atresia?
Baby isn’t born with valve to pulmonary artery, lack of blood flowing to lungs for oxygenation.

-baby will have: cyanosis, poor feeding, sob, and extreme sleepiness.

What is transposition of the great arteries?
Aorta and Pulmonary artery are switched

-blood from the body enters right side of heart and pumped directly back out to body. Blood from lungs pumps to left ventricle then back to lungs.

Ventricle Septal Defect (VSD)
flaw in the septum that divides the two ventricles of the heart

  • baby is usually asymptomatic
  • will hear holosystolic murmur

Tx: small will close, large needs surgery.

What is Tetralogy of Fallot?
Tetralogy means 4

1) pulmonary stenosis
2) overriding aorta (receives blood from left and right ventricle)
3) RVH (right ventricle hypertrophy)
4) VSD (ventricular septal defect)

Most common cyanotic congenital heart disease in children

patent ductus arteriosus (PDA)
passageway (ductus arteriosus) between the aorta and the pulmonary artery remains open (patent) after birth

Symptoms:
wide pulse pressures
Enlarged heart
Respiratory issues

What is the treatment for patent ductus arteriosus?
NSAID- decrease prostaglandin which allow duct to close

Surgery

What are the risk factors for hypertension in children and teens?
-being obese
-Exposure to secondhand smoke

In evaluating a nine-year-old child with a healthy BMI during a well visit, a comprehensive cardiovascular evaluation should be conducted by the following methods:
-obtaining a fasting lipid profile
-assess diet and physical activity

At what age is it appropriate to recommend dietary changes to parents if overweight or obesity is a concern?
12 months

What are the risk factors for type two diabetes in children and teens?
-Hyperinsulinemia
-Abnormal weight to height ratio
-BMI >85 percentile
-Native American ancestry

When is screening for children with a known risk factor of type 2 diabetes recommended? And should Be repeated how often?
Onset of puberty or Age 10 which ever comes sooner

If normal, repeat at minimum of 3 year intervals, more frequently if BMI increasing.

A 12 year old boy presents with his mother post pre-diabetes diagnosis. Upon reviewing the chart the patient is at 85 percentile For height and weight. The mother ask what they can do to prevent the patient from moving to full-blown diabetes. What would be your response?
-patient should be encouraged to participate in at least 60 minutes of moderate to vigorous physical activity daily
-muscle and bone strength training at least three days a week
-Decrease sedentary behavior

What is an A1c target for most children and adolescents with type two diabetes treated with oral agents?
<7%

Pre-diabetes in children is defined as?
Impaired fasting glucose 100-125
-random glucose 140-199

What are the risk factors for dyslipidemia in children?
Family history of lipid abnormalities
Family history of type two diabetes mellitus

At what age does screening cholesterol levels in children with one or more risk factors begin at?
Age 2

A 17-year-old male presents post lipid labs, his total cholesterol Level is 150. What education would you provide?
This is an acceptable total cholesterol level (<170),

Will recheck in 3 years

Low birthweight and poor infant growth are risk factors for type two diabetes. True or false?
True

When should body mass index be measured?
At age 2

A mother of a 14-month-old presents with concerns for her child being overweight. When you review the growth chart the patient is in the 99th percentile. What recommendation can be made?
Patient can be switched to reduced fat milk instead of whole milk.

A seven-year-old boy who plays football presents with complaints of left knee pain. He denies any recent trauma but reports pain is worse with activity or when applying pressure to the knee over tibial tubercle. He endorses there has been intermittent swelling. What is the dx and tx?
Osgood-schlatter disease- common in athletic boys.

TX:
-RICE
-Patellar tendon strap
-Stretching before activity
-NSAIDs

  • Should resolve spontaneously in 12 to 18 months

3 year old female presents in March with diarrhea. Mother states started with slight fever and vomiting which resolved. Mother concerned with dehydration. What is the diagnosis and education?
Rotavirus

-usually last 3-8 days
-rotavirus vaccine
-hand hygiene
-s/s of dehydration

What test is used to confirm iron deficiency anemia?
Serum ferritin

Spoon-shaped Fingernails (koilonychia) is a presentation for which type of anemia?
Iron deficiency deficiency list it

A 13-year-old female presents with the following labs: decreased Hgb & Hct, decreased ferritin, Decreased serum iron, increased TIBC. What is the most likely type of anemia?
Iron deficiency anemia

A three year old female presents with her parents for a well visit. Upon exam you note everything within normal limits. The parents mention that the patient is eating odd things out of the yard like sticks and dirt. They question whether this is a normal three-year-old behavior or something to be concerned about. What is your response?
This may be pica a sign of iron deficiency anemia. We will get a fingerstick to be sure.

What are signs and symptoms of iron deficiency anemia?
Headache, palpitations, dyspnea on exertion, pallor, chronic fatigue, pale conjunctiva, murmurs, tachycardia, heart failure

*cheilits (fissures at corner of mouth)
*Koilonychia ( spooning of fingernails)

What is the gold standard diagnostic test for thalassemia?
Hemoglobin electrophoresis

What is amblyopia?
-Amblyopia, also known as lazy eye, is a condition in which one or both eyes do not develop normal vision due to various factors that cause the visual part of the brain to function abnormally. This weakens the eye and can lead to long- term vision problems.

TX: If recognized early, amblyopia generally responds well to treatment. Amblyopia therapy can include glasses, patching, eye drops, and sometimes surgery.

What is epiphora?
-Epiphora is the term for excessive tearing. Childhood epiphora is often noted soon after birth, but can be acquired later.

CAUSES: When noted during infancy, it is usually due to blockage of the tear drainage system.
-rare causes: glaucoma and ocular surface disease

TX: This type of tearing often improves spontaneously by 6 to 12 months of age. Medical treatment includes tear sac massage and eye drops, but if tearing persists, surgical probing of the drainage system may be required.

A 12 month old presents for a well visit, upon exam you notice a cloudiness to the eye lens along with a white area in the pupil with misalignment. What is the diagnosis and treatment?
Pediatric Cataract

*can cause blurry vision

TX:
Pediatric cataracts that significantly obstruct vision require surgery. Patients subsequently require treatment with eyeglasses, bifocals, or contact lenses, and eye-patching.

*Often, pediatric cataracts result in some degree of lazy eye (amblyopia) and strabismus as well.

A 9 year old female presents with complaint of headaches every day X 5 days. When asked to describe the pain patient reports they are sharp, stabbing pain to the left side of her head. When asked about accompanying symptoms patient reports left eye tearing, and runny nose. What education and treatment would you provide?
This is a cluster headache, take NSAIDs

Headaches to be concerned about:
-Wake your child from sleep
-Worsen or become more frequent
-Change your child’s personality
-Follow an injury, such as a blow to the head

  • Feature persistent vomiting or visual changes
  • Are accompanied by fever and neck pain or stiffness

Risk factors for headaches:
-Girls after they reach puberty
-Children who have a family history of headaches or migraines
-Older teens

Genu valgum (knock knees)
This is a condition in which the knees touch but the ankles do not. Can be normal in children up to age 6.

*braces for angles more than 15 degrees

muscular dystrophy
group of hereditary diseases characterized by degeneration of muscle and weakness

S/S: hypotonia, muscle weakness, contractures, motor deficit.

DX: CK, muscle biopsy, MRI, EMG. PT, OT, nutrition

*steroids can slow progression, ultimately deadly.

Leave a Comment

Scroll to Top